Final MBE stuff to know Flashcards

1
Q

Employee stops with company car at dry-cleaner and gets into accident because of driving negligently. Employer liable?

Yet
An electrician was employed by an electrical services company that had contracts with a number of large office and condominium buildings to provide emergency electrical services and repairs at any hour of the day or night. Hence, he was required to be “on call” 24 hours a day and to drive his company van, which had all of his tools, to his home each night. One afternoon, the electrician left the company’s office at 4 p.m. as usual. However, when he left the main highway, he did not turn left toward his home but instead turned right toward the supermarket a few blocks away to pick up some items for dinner. While leaving the supermarket parking lot, the electrician drove negligently and struck a pedestrian. The pedestrian suffered serious injuries and required several operations and a lengthy hospital stay. The pedestrian filed suit against the company for $100,000.

A

An employer will be vicariously liable for tortious acts committed by its employee only if the tortious acts occur within the scope of the employment relationship. Here, although the employee was using the company car, she was not conducting any business for the delivery company. Her use of the car to go grocery shopping was a personal errand outside the scope of her employment for which the delivery company is not vicariously liable.

[2]
Ordinarily, an employee heading home after work is no longer within the scope of employment. Here, however, the electrician was required to be “on call” 24 hours a day, and was required to drive the company van to his home so he would be ready to provide emergency service whenever a call would come in.

Most likely, then, the electrician was still within the scope of his employment when he was driving the van home. The next issue is whether his deviation from his route home took him outside the scope of his employment. Most courts today consider the foreseeability of the deviation to be the most important factor in determining whether the employee was still within the scope of employment or was on a “frolic” of his own. Thus, minor deviations in time and geographic area from the employer’s business are still within the scope of employment because they are foreseeable. Here, the electrician’s deviation of a few blocks from his normal route home to pick up some groceries was not a substantial enough departure from his employment purposes so as to be unforeseeable, and therefore the company can be held vicariously liable for the electrician’s negligence.

How well did you know this?
1
Not at all
2
3
4
5
Perfectly
2
Q

A chef agreed in writing to lease a restaurant from the owner of the property. The term of the tenancy was two years, and rent was payable in monthly installments at the beginning of each month. At the end of the second year, there had been no discussions between the chef and the owner regarding renewal or termination. The chef did not vacate the premises at the end of the term; instead, she sent a check for the next month’s rent to the owner. The owner cashed the check after the term had expired but informed the chef that his acceptance of the check did not mean that he was going to renew the lease or let the chef stay. At the end of that month, the owner seeks advice on whether he can evict the chef.

How should the owner be advised to proceed?

Press Enter or Space to submit the answer

A The owner must give the chef a full 30 days’ notice before beginning eviction proceedings because a month-to-month periodic tenancy has been created.

Correct
B The owner may begin eviction proceedings as soon as the additional month has expired.

Incorrect
C The owner may not evict the chef for 11 months and must give six months’ notice before beginning eviction proceedings because a year-to-year periodic tenancy has been created.

D The owner may not evict the chef for 11 months but need not give any notice prior to eviction because a tenancy for years for a term of one year has been created.

A

Landlord keeps rent for one month, be he did not decide to create a periodic tenancy.

The owner may begin eviction proceedings at any time. When a tenant continues in possession after the termination of her right to possession, the landlord has two choices of action: He may treat the hold-over tenant as a trespasser and evict her under an unlawful detainer statute, or he may, in his sole discretion, bind the tenant to a new periodic tenancy, in which case the terms and conditions of the expired tenancy apply to the new tenancy. Here, while the owner accepted the check sent by the chef, he informed her that he was not electing to bind her to a new tenancy. The owner may keep the check because he is entitled to rent for the month that the chef was a hold-over tenant, but at the end of that month he has the right to evict her because no periodic tenancy was created and the chef’s right to possession has terminated.

(A) is incorrect because the owner did not elect to create a periodic tenancy. Furthermore, had he done so, the tenancy would have been a year-to-year tenancy rather than a month-to-month tenancy because it is a commercial lease for more than one year, rather than a residential lease.

(C) is incorrect because, as discussed above, the owner did not elect to create a periodic tenancy when the chef held over.

How well did you know this?
1
Not at all
2
3
4
5
Perfectly
3
Q

The plaintiff sued the defendant, who had constructed the plaintiff’s house, for breach of warranty of habitability. At trial, in cross-examination of the plaintiff, the defendant’s attorney asked whether the plaintiff had sued another contractor 30 years earlier, claiming similar defects in another house built for the plaintiff. The question was not objected to and the plaintiff answered that she had had some “water problems” with the first house she ever purchased, but no suit was filed.

The defendant then called as a witness the contractor of 30 years earlier to testify that the plaintiff had brought suit against him for defects in the earlier house, many of which were like those now claimed to be found in the home the defendant built, but that the case was settled without trial.

Should the trial court rule that the witness’s offered testimony is admissible?

A Yes, as proper impeachment because the plaintiff will have an opportunity to explain or deny the witness’s statement.

B Yes, because the plaintiff failed to object to the defendant’s questions on cross-examination relative to the prior suit.

C No, because the best evidence of the former suit is the court record.

D No, because its probative value is substantially outweighed by the danger that it will confuse the issues and waste time.

A

The trial court should rule the witness’s testimony inadmissible because its probative value is substantially outweighed by the danger that it will confuse the issues and waste time. Where a witness makes a statement not directly relevant to the issues in the case, the rule against impeachment (other than by cross-examination) on a collateral matter applies to bar the opponent from proving the statement untrue either by extrinsic contradictory facts or by a prior inconsistent statement. The purpose of the rule is to avoid the possibility of unfair surprise, confusion of issues, and undue consumption of time.

An issue is considered collateral if it would not be admissible other than to contradict the testimony.

Evidence that a person has previously filed similar claims is generally inadmissible to show the invalidity of the present claim. At best, this evidence shows the plaintiff’s tendency toward litigation. Unless there is evidence that the previous claim was false, the probative value of such evidence is deemed outweighed by the risk of confusion of the issues. Because the prior suit would not be the subject of proof independent of impeachment, it is a collateral matter, and extrinsic evidence, such as the witness’s testimony, is inadmissible.

How well did you know this?
1
Not at all
2
3
4
5
Perfectly
4
Q

The federal government contracted with a number of communications utilities to install fiberoptic communication lines between major federal offices across the country. The utilities, which maintained ownership of the lines, contracted with the federal government to install the lines on a “cost plus fixed fee” basis, whereby all installation costs would be reimbursed by the government. One such line was installed in a state’s capital city, where the Department of the Interior maintained its western regional office. The state imposes a tax on the installation of all communication lines in the state, including fiberoptic cable lines. It seeks to impose the tax on the line running to the federal office.

Will the state be permitted to impose the tax?

A Yes, because the tax is indirect and nondiscriminatory.

B Yes, because the tax is a valid exercise of state power under the Tenth Amendment.

C No, because the tax burdens the activities of the federal government.

D No, because the activity taxed involves interstate commerce.

A

The state may impose a tax on the fiberoptic line. A state tax levied directly against the property or operation of the federal government without the consent of Congress is invalid. However, nondiscriminatory, indirect taxes on the federal government or its property are permissible if they do not unreasonably burden the federal government. Because this tax is not levied directly against the government, but rather against the provider of a service that the government is obtaining, and is levied on all communications lines in the state, the tax is valid. The fact that the economic burden of the tax will ultimately be borne by the government under the “cost plus” contract does not invalidate the tax.

(C) is wrong because not every state tax that burdens the federal government is invalid. A nondiscriminatory tax on a service provided to the federal government by a private entity does not appear to be an unreasonable burden on the operation of the federal government.

How well did you know this?
1
Not at all
2
3
4
5
Perfectly
5
Q

On a wholly random basis, a state agency has given a few probationary employees who were not rehired at the end of their probationary period a statement of reasons and an opportunity for a hearing, but the agency has very rarely done so. No statute or rule of the agency required such a statement of reasons or a hearing. The employment of a probationary employee was terminated without a statement of reasons or an opportunity for a hearing. The agency did not even consider whether it should give him either.

Will a suit by the employee requesting a statement of reasons and a hearing be successful?

A Yes, on the grounds that failure to give the employee reasons and an opportunity for a hearing constituted a bill of attainder.

B Yes, on the grounds that an agency’s inconsistent practices, even if unintentional, deny adversely affected persons the equal protection of the law.

C No, because the employee does not have a right to be rehired that is protected by procedural due process.

D No, because the conditions of state employment are matters reserved to the states by the Tenth Amendment.

A

Employee, without a statute is an employee at will and has no property interest and therefore no procedural rights.

The employee’s suit will be unsuccessful because he has no right to a hearing here because he has no life, liberty, or property interest at stake. The Due Process Clause requires a hearing only when a life, liberty, or property interest is at stake. The employee clearly is not at risk of losing his life or liberty, and the Supreme Court has made clear that neither is a property interest involved here. To have a property interest in continued government employment, there must be a statute, regulation, contract right, or clear policy that the employee can be dismissed only for cause. Absent such a right to employment, the employee is an at-will employee and may be terminated without a hearing. Here, there was no law, contract, or policy giving the employee a right to a job absent cause for firing him. Therefore, no hearing was required. (A) is incorrect because bills of attainder involve criminal or otherwise punitive measures inflicted without judicial trial. Nothing here indicates that the employee is being punished; rather he is not being retained as an employee. (

How well did you know this?
1
Not at all
2
3
4
5
Perfectly
6
Q

What happens when D lawyer sends a notice of deposition without subpoena to a non-party and the nonparty does not show up? Can P lawyer recover expenses of sending its lawyer to the deposition?

A

The plaintiff may recover reasonable expenses because the defendant’s attorney did not serve a subpoena on the witness. When a party who notices a deposition fails to serve a subpoena on a nonparty deponent who then does not appear, the opposing party may recover reasonable expenses for attending, including attorney’s fees.

How well did you know this?
1
Not at all
2
3
4
5
Perfectly
7
Q

Can a landlord burn his own building that is currently being rented?

A

NO The requirement that the building be “of another” pertains to possession rather than ownership. Thus, a landlord could be guilty of arson for burning down his own building if his tenants were in possession of it rather than him;

How well did you know this?
1
Not at all
2
3
4
5
Perfectly
8
Q

Implied in fact contracts

A professor who was an expert on American history conducted full-day tours through the historic sites of Philadelphia. The professor’s fee for his services was $105, which did not include the entrance fees for several of the historical sites. A law student took a day off school and “hung around” the Liberty Bell monument, where the professor’s tour started. That day the professor was conducting 27 persons on the tour. Most of the participants had paid in advance, but the professor was holding up a sign with information about the tour and handing out brochures, one of which the law student took. The professor accepted a few additional participants who signed up on the spot, but the law student was not among them. All day long, the law student hung around at the fringe of this group, paying the entrance fees separately but following the group through the different historical sites. However, he always positioned himself close enough to the professor’s group so that he could hear virtually every word of the professor’s lecture, although the law student did not ask the professor any questions. The law student signed his name and address on the register at Independence Hall. The professor noted this and took down the information. Two days after the tour concluded, the law student received a bill from the professor in the amount of $105.

How much will the law student most likely be required to pay the professor?

A $105, because that is the contract price for the tour.

B $105, because the amount of the contract was less than $500, making the Statute of Frauds inapplicable.

C $105, if that is a reasonable fee for the lectures based on the professor’s expertise.

D Nothing, because the historical sites were open to the public and the law student paid his own way.

A

The law student will probably be required to pay the professor $105 under an implied-in-fact contract. An implied-in-fact contract is a contract formed by manifestations of assent other than oral or written language, i.e., by conduct. Even if there is no subjective “meeting of the minds,” the parties will be bound if their conduct objectively appears to manifest a contractual intent. Where an offeree silently takes the benefit of offered services with reasonable opportunity to reject them and reason to know that they were offered with the expectation of compensation, the offeree’s inaction may constitute an acceptance. [Restatement (Second) of Contracts §69(1)(a)] Here, the student’s silence in the face of the professor’s offer and his conduct in staying within earshot of the group is a sufficient objective manifestation of contractual intent for the court to find an implied-in-fact contract. Hence, a court will probably allow the professor to recover the contract price. (B) is wrong because it states the wrong rationale. The Statute of Frauds would not be applicable even if the cost of the tour were over $500; the $500 provision of the Statute of Frauds is applicable only to the sale of goods. (C) is wrong because it states a restitutionary remedy available in a quasi-contract action. A quasi-contract action for restitution is a legal remedy to prevent unjust enrichment where an enforceable contract is not present, and allows the claimant to recover the reasonable value of the benefits that he rendered to the other party. While the professor probably could pursue a quasi-contract action for restitution because he rendered services with a reasonable expectation of being compensated, he is not limited to that remedy because he can establish an implied-in-fact contract. Hence, he can recover the contract price for the tour without having to establish that it was a reasonable fee for the lectures.

How well did you know this?
1
Not at all
2
3
4
5
Perfectly
9
Q

Nice little RAP q for you

Fifteen years ago, a homeowner executed his will, devising his home “to my surviving widow for life, remainder to such of my children as shall live to attain the age of 30 years; but if any child dies under the age of 30 years survived by a child or children, such child or children shall take and receive the share which his, her, or their parent would have received had such parent lived to attain the age of 30 years.” At the date of writing his will, the homeowner was married to an actress, and they had two adult daughters. The actress died 10 years ago, and the homeowner married a dancer two years later. At his death last year, the homeowner was survived by the dancer and three children, the two daughters from his marriage to the actress, and a son. The son, who is six years old, was the homeowner’s child by the dancer. The jurisdiction recognizes the common law Rule Against Perpetuities unmodified by statute.

What is the result of the application of the Rule?

A The remainder to the children and to the grandchildren is void, because the homeowner could have subsequently married a person who was unborn at the time he executed his will.

B The remainder to the children is valid, but the substitutionary gift to the grandchildren is void, because the homeowner could have subsequently married a person who was unborn at the time he executed his will.

C The gift in remainder to the daughters or their children is valid, but the gift to the son or his children is void.

D The remainder to the children and the substitutionary gift to the grandchildren are valid.

A

The gifts are valid under the Rule. The homeowner’s will created a life estate in the dancer, contingent remainders in the class consisting of the homeowner’s children (contingent upon their attaining age 30), and contingent remainders in the class consisting of any children of the homeowner’s children (contingent on their surviving their parent, and the parent dying before attaining age 30). There are two keys to understanding the question. The first is that a will speaks at death, no matter when it was executed. Here, the homeowner’s will became an effective conveyance only when he died last year. The second key is that the grandchildren (i.e., the children of the daughters or the son) do not themselves have to survive to any particular age to take their gifts. The wording of the question is somewhat confusing on this point, but it is clear when read carefully. Because there are two future interests in the question, each must be analyzed separately under the Rule Against Perpetuities. The gift to the homeowner’s children is a class gift, and the Rule makes class gifts entirely void unless it is certain that the gift will vest or fail as to all members of the class within the perpetuities period. However, it is clear that this will be true here. The three children (the daughters and the son) are all alive when the will speaks. Hence, they are all lives in being. (If the dancer had been pregnant when the homeowner died, that child, when born, would also have been considered a “life in being” as of the homeowner’s death.) The gift is certain to vest as to each of the homeowner’s children when each reaches age 30, which is obviously within each child’s lifetime. Likewise, if one of the children dies before age 30, his or her interest will fail; again, that is certain to happen within his or her lifetime. Because this is so, the class gift to the children of the homeowner is certain to vest or fail as to each member within “lives in being.” The gift is therefore valid under the Rule. It is not even necessary to add the 21-year period as permitted by the Rule. As to the class gift to the grandchildren of the homeowner, a similar analysis follows. If any grandchild’s interest ever becomes vested, it will do so immediately on the death of that grandchild’s parent (one of the daughters or the son) prior to reaching age 30. Because those three persons are “lives in being” at the homeowner’s death, the grandchildren’s interests are certain to vest or fail in every case at the end of a life in being. Again, it is not necessary to add the 21-year period as permitted by the Rule.

(A) is wrong because the time of execution of the will is irrelevant; it is the date of the testator’s death that commences the running of the perpetuities period.

(B) is wrong for the same reason.

(C) is wrong for the reasons discussed above.

How well did you know this?
1
Not at all
2
3
4
5
Perfectly
10
Q

When can a joint tortfeasor ask for indemnification?

A

Generally, a joint tortfeasor may recover indemnification from another joint tortfeasor where there is a considerable difference in the degree of fault. Here, the engineer, the person whose improper design actually caused the girl’s injuries, is a “more wrongful” tortfeasor than the manufacturer.

How well did you know this?
1
Not at all
2
3
4
5
Perfectly
11
Q

What do you get for partial performance?

A

In such cases where the builder breaches after partially performing, the owner of the land is entitled to the cost of completion plus reasonable compensation for any delay in performance. Courts generally allow the builder to offset or recover for work performed to date to avoid the unjust enrichment of the owner. Hence, the unpaid installments should be deducted.

How well did you know this?
1
Not at all
2
3
4
5
Perfectly
12
Q

Can a defeasible fee simple owner take minerals out?

A

Yes.The owner of a defeasible fee has the same right to possession and privileges of use as the owner of a fee simple absolute.

How well did you know this?
1
Not at all
2
3
4
5
Perfectly
13
Q

A lawyer was appointed as an administrative judge to review claims against the federal government made by Native Americans under a congressional statute. For 20 years, the lawyer heard, reviewed, and arbitrated disputed claims made against the government by various Indian tribes and their citizens. When the lawyer found a claim to be valid, he would make a recommendation to the Bureau of Indian Claims that the claim be paid. If the lawyer found the claims to be without merit, or if the Bureau decided against his recommendation, the claimant would have the right to bring suit in a federal court. Last year, a presidential commission recommended the abolition of the Bureau of Indian Claims as a cost-cutting measure. Congress acted on this recommendation and repealed the statute. The lawyer was offered a position as an attorney in the Department of Transportation, but he turned it down and brought suit against the government.

What is the likely result of this suit?

A The lawyer prevails, because it violates the doctrine of separation of powers for the executive branch to interfere with a congressional act by recommending its repeal.

B The lawyer prevails, because it violates the Constitution to terminate the tenure of a federal judge during good behavior.

C The government prevails, because it established the lawyer’s position and it can terminate it at will.

D The government prevails, because the lawyer had no judicial discretion or powers in his position with the Bureau.

A

The government will most likely prevail because the lawyer had no judicial discretion or powers in his position. Under Article III of the Constitution, a federal judge is protected from termination of tenure during good behavior. This necessarily requires that a person who seeks protection under this provision be able to show that he is a federal “judge.” From the facts, the lawyer was clearly no more than an administrative hearing officer, without discretion or power. Thus, he would not be a judge within the meaning of this article, and its provisions would not apply to him. Therefore (B) is wrong. (A) is wrong because anybody can recommend that Congress enact or repeal a statute. Just because an executive branch’s commission does so does not mean that there is a violation of the separation of powers doctrine. (C) is factually incorrect and does not explain the proper reason for the result.

How well did you know this?
1
Not at all
2
3
4
5
Perfectly
14
Q

What happens if lawyer accidentally sends priviledge information?

A

If a trial party inadvertently discloses privileged material to an opposing party, it may still invoke a claim of privilege by notifying the opposing party of the disclosure and the basis for the claim of privilege. Once notified, the opposing party must promptly return, sequester, or destroy the specified information and take reasonable steps to retrieve the material if it disclosed it to others. The opposing party also may not use or disclose the privileged material until the claim is resolved.

How well did you know this?
1
Not at all
2
3
4
5
Perfectly
15
Q

Larceny or continuing trespass?

A woman was in the process of moving out of the apartment that she had shared with a roommate. She collected numerous items of hers from her roommate’s room that the roommate had borrowed. As she was leaving the apartment, she grabbed what she believed to be her laptop computer, which her roommate had often borrowed. Because it was an older, slower machine, she planned to trade it in for a different model at a computer resale store. She noticed that the laptop was much lighter than usual, but she reasonably attributed this to her diligence in following a weight-lifting regimen at her gym. When she arrived at the computer store, she discovered that she had taken a brand new, state-of-the-art laptop that her roommate had recently purchased. She then kept the laptop rather than buying a new one.

Is the woman guilty of common law larceny?

A No, because she mistakenly believed that the computer she had picked up was hers.

B No, because her mistake as to whose computer she had picked up was reasonable.

C Yes, because she intended to keep the computer when she took it.

D Yes, because she decided to keep the computer when she discovered the mistake.

A

The woman is not guilty of common law larceny of the computer because her mistake prevented her from having the requisite mens rea for larceny. Larceny requires the intent to permanently deprive another of her interest in the property taken. The woman did not have such intent, given that she believed that the computer was her own and that her roommate had no possessory interest in it. Therefore, she did not have the intent required for larceny. (B) is wrong because the woman’s mistake need not have been reasonable. When mistake is offered to negate the existence of general intent or malice, it must be a reasonable mistake. However, any mistake of fact, reasonable or unreasonable, is a defense to a specific intent crime, and larceny is a specific intent crime. (C) is wrong because, as stated above, she did not have the intent to deprive her roommate of her roommate’s computer; her mistake negates such intent.

(D) is wrong because the “continuing trespass” doctrine is inapplicable. Although larceny generally requires the intent to deprive another person of her interest in the property at the moment of taking, the continuing trespass doctrine provides that if a defendant takes property with a wrongful state of mind, but without the intent to steal, and then he later forms the intent to steal it, the trespass involved in the initial wrongful taking is regarded as “continuing” and the defendant is guilty of larceny. However, this doctrine has no application if the defendant’s initial taking of the property, although trespassory, was not motivated by a wrongful state of mind. Here, the woman took her roommate’s computer as a result of an innocent mistake. Even though she decided to keep the computer, she will not be guilty of larceny because her initial taking was done with an innocent state of mind.

How well did you know this?
1
Not at all
2
3
4
5
Perfectly
16
Q

An entrepreneur purchased a piece of undeveloped land with plans to build a luxury spa. He financed the purchase with a loan from a bank, secured by a mortgage on the land. The land contained a mineral hot spring. The entrepreneur hired builders to harness the spring’s water into a soaking pool. The spa was not a success and the entrepreneur ran out of money. He entered into a contract that purported to transfer his inventory and all his interests in the soaking pool to a buyer by a document that was sufficient as a bill of sale to transfer personal property but was insufficient as a deed to transfer real property. The bank soon after foreclosed on its mortgage and the land was sold at auction to a bidder, who took title in fee simple.

Who has title to the soaking pool?

A The bidder, as fee simple owner of the land.

B The buyer, as purchaser of the soaking pool under the bill of sale.

C The person who owns the water rights as an incident thereto.

D The entrepreneur, as the builder of the soaking pool.

A

Title to the soaking pool resides in the bidder, the fee simple owner of the land. Under the concept of fixtures, the soaking pool was converted from personalty into realty. The soaking pool is an accessory to the land and passes with the ownership of the land. (B) is incorrect because the document purporting to transfer the entrepreneur’s interest in the soaking pool to the buyer was insufficient to transfer real property. (C) is an incorrect statement of the law. (D) is incorrect because the soaking pool, as an accessory to the land, belongs to the owner of the land.

How well did you know this?
1
Not at all
2
3
4
5
Perfectly
17
Q

An owner conveyed her parcel of land to her church “for the life of my son, and from and after the death of my said son to all of my grandchildren and their heirs and assigns in equal shares; provided that the church shall use the premises for church purposes only.” In an existing building on the property, the church immediately began to conduct religious services and other church activities. Subsequently, the church granted a construction company the right to remove sand and gravel from a one-half acre portion of the property on payment of a royalty. The construction company has regularly removed sand and gravel since and paid a royalty to the church. The church has continued to conduct religious services and other church activities on the property. All four of the living grandchildren of the owner, joined by a guardian ad litem to represent unborn grandchildren, instituted suit against the church and the construction company seeking damages for the removal of sand and gravel and an injunction preventing further acts of removal. There is no applicable statute.

Which of the following best describes the likely disposition of this lawsuit?

A The injunction and damages should be granted, because the interest of the church terminated with the first removal of sand and gravel.

B The injunction should be granted, and damages should be recovered but impounded for future distribution.

C The injunction should be granted, but damages should be denied because the owner and her son are not parties to the action.

D The injunction should be denied, but damages should be awarded.

A

Both an injunction and damages should be ordered. The church has a life estate pur autre vie, and a life tenant as a general rule is not entitled to consume or exploit natural resources on the property; this constitutes affirmative (voluntary) waste that injures the interests of the future interest holders. Any award of damages will be held until the class gift to the grandchildren closes at the son’s death. (A) is wrong because the church’s action did not terminate its interest. The “provided that” language creates a condition subsequent. An estate subject to a condition subsequent does not terminate automatically on the happening of the condition. To terminate, the grantor must exercise a right of entry, and here no right of entry was reserved. (C) is wrong because it is entirely unnecessary for the owner and her son to be parties, because neither of them has any interest in the land. The owner has given up her interest entirely, and the son is present in the conveyance only to serve as a measuring life for the life estate; he owns no interest in the land itself. (D) is wrong because the injury to the land is permanent and therefore should be prevented by an injunction.

How well did you know this?
1
Not at all
2
3
4
5
Perfectly
18
Q

Three American drivers were involved in a three-car accident in Canada. One of them intends to file a tort action against the other two in a United States district court. The plaintiff resides in the District of State A. One defendant resides in the District of State B, and the other defendant resides in the District of State C.

In which federal district(s) is venue proper?

A The District of State A only.

B The District of State B and the District of State C only.

C The District of State A, the District of State B, and the District of State C.

D Venue is not proper in any district because actions arising from an automobile accident outside of United States territory may not be maintained in United States courts.

A

Where is venue proper summary

Federal venue in civil actions is proper in (i) the district where any defendant resides, if all defendants are residents of the state in which the district is located; and (ii) the district in which a substantial part of the events or omissions giving rise to the claim occurred. Here, the defendants’ residences are not a basis for proper venue because the defendants do not reside in the same state, and the events giving rise to the action occurred in Canada, not in any United States judicial district. When there is no other district in which an action may be brought, venue is proper in any district in which any defendant is subject to the court’s personal jurisdiction with respect to the action. Each of the defendants is subject to personal jurisdiction in the district where he resides, thereby rendering each of those districts proper.

How well did you know this?
1
Not at all
2
3
4
5
Perfectly
19
Q

A mechanic sued his former employer in federal court, claiming that the employer had discharged him because of his age in violation of federal law. The employer answered, denying the claims and promptly moving for summary judgment. In support of the motion, the employer attached the mechanic’s employment evaluations for the past three years, which rated his skills and performance as poor and culminated in a recommendation for his discharge.

What is the mechanic’s best argument to defeat the summary judgment motion?

A The allegations in the complaint conflict with the mechanic’s employment evaluations, raising a genuine dispute as to material facts.

B The employer cannot rely in his motion on matters outside the pleadings.

C The essential facts are unavailable to the mechanic and therefore discovery is required.

D The motion was filed before the close of discovery.

A

(C) is correct. If the mechanic (the nonmovant) shows by affidavit or declaration that he cannot present facts essential to justify his opposition to the summary judgment motion, Rule 56(d) authorizes him to ask the court to defer action or deny the motion to allow time to obtain affidavits or declarations or to take discovery. The employer moved for summary judgment right after answering and before any discovery. That timing would support defeating the summary judgment motion at this time.

(A) is incorrect. Under Rule 56(c), a party asserting that a fact is genuinely disputed must support the assertion by citing particular parts of the record, including affidavits or declarations, stipulations, or discovery materials. The mechanic cannot simply rely on the complaint allegations to rebut the employer’s evidence but must support his factual position with his own evidence that a factual dispute exists. If he cannot do so, Rule 56(d) authorizes him to ask the court to defer action or deny the summary judgment motion to allow time to obtain affidavits or declarations or to take discovery.

(B) is incorrect. The function of a summary judgment motion is to allow additional evidence outside the pleadings to show that there is no genuine dispute of fact and that the movant is entitled to judgment as a matter of law. Rule 56(c)(1) enumerates the types of materials that the moving party may use to support a summary judgment motion, including documents. If the mechanic (the nonmovant) shows by affidavit or declaration that he cannot present facts essential to justify his opposition to the summary judgment motion, Rule 56(d) authorizes him to ask the court to defer action or deny the motion to allow time to obtain affidavits or declarations or to take discovery.

(D) is incorrect. The fact that a summary judgment motion is filed before the close of discovery does not require the court to deny it. Under Rule 56(b), a party may file the motion at any time until 30 days after the close of all discovery. The problem here is that the employer filed the motion before discovery commenced, thus providing the mechanic an argument to defer action or deny the motion under Rule 56(d).

How well did you know this?
1
Not at all
2
3
4
5
Perfectly
20
Q

A builder contracted in writing to construct a small greenhouse on a homeowner’s property for $20,000, payable upon completion. After the builder had spent $9,000 framing the greenhouse and an additional $1,000 for materials not yet incorporated into the greenhouse, the homeowner wrongfully ordered the builder to stop work.

The builder then resold the unused materials that he had already purchased for the greenhouse to another contractor for $1,000. At the time the homeowner stopped the work, it would have cost the builder an additional $5,000 to complete the project. The partially built greenhouse increased the value of the homeowner’s property by $3,000.

In a suit by the builder against the homeowner, how much is the builder likely to recover?

A $3,000, the increase in the value of the homeowner’s property.

B $10,000, the total cost expended by the builder at the time of the breach.

C $14,000, the total cost expended by the builder ($10,000) plus the builder’s expected profit ($5,000), minus the loss avoided by the resale of the unused materials ($1,000).

D $15,000, the contract price ($20,000) minus the costs saved by the breach ($5,000).

A

(C) is correct. The builder is likely to recover $14,000. In a construction contract, if the property owner breaches the contract during construction, the builder is entitled to any profit he would have derived from the contract plus any costs he has incurred to date. If the builder has mitigated his damages, any losses that are avoided must be subtracted from this amount.

(A) is incorrect. In a construction contract, when the property owner breaches before the construction is completed, the builder’s damages are not measured by the increase in value of the homeowner’s property.

(B) is incorrect. The builder is also entitled to the profit he would have made if the contract had been performed.

(D) is also incorrect. The formula for awarding a builder damages for a breach during a construction contract can also be stated as the contract price minus the cost of completion, which would be $15,000. However, this answer fails to account for the $1,000 of damages the builder avoided by reselling the unused materials.

How well did you know this?
1
Not at all
2
3
4
5
Perfectly
21
Q

man filed a federal diversity action against a bus company, seeking damages for injuries he had sustained in an accident while riding a bus owned by the company. The man demanded a jury trial.

After the parties’ attorneys examined the prospective jurors and exercised their challenges, six jurors and two alternate jurors were chosen. During the trial, two jurors became ill and were replaced by the alternate jurors. At the conclusion of the trial, a third juror also became ill, and the court excused that juror.

The parties’ attorneys stipulated to the return of a verdict from a five-person jury. The jury then deliberated and returned a verdict for the company. The man timely filed a motion for a new trial, arguing that the five-person jury was not large enough to return a verdict.

Should the court grant the motion?

A No, because the court properly excused the three jurors due to illness.

B No, because the parties stipulated to a verdict from a jury of fewer than six jurors.

C Yes, because there must be at least six jurors on a federal civil jury.

D Yes, because there must be at least 12 jurors on a federal civil jury.

A

B) is correct. A jury must be composed of at least six jurors at the beginning of the trial. If the number drops below six (for example, as here, when jurors became ill), a mistrial results, unless the parties agree to a lesser number of jurors. Because the parties stipulated to fewer jurors, the issue is waived. (A) is incorrect because even though the jurors were properly excused, a lesser number of jurors cannot return a verdict unless the parties agree. (C) is incorrect because the parties may stipulate to fewer jurors. (D) is incorrect. A jury must be composed of at least six jurors, not 12, and the answer does not address the possibility of a stipulation.

How well did you know this?
1
Not at all
2
3
4
5
Perfectly
22
Q

Eeven years ago, a man conveyed vacant land by warranty deed to a woman, a bona fide purchaser for value. The woman did not record the warranty deed and did not enter into possession of the land.
Five years ago, the man conveyed the same land to a neighbor, also a bona fide purchaser for value, by a quitclaim deed. The neighbor immediately recorded the quitclaim deed and went into possession of the land.

Two years ago, the neighbor conveyed the land to a friend, who had notice of the prior conveyance from the man to the woman. The friend never recorded the deed but went into immediate possession of the land.

The jurisdiction has a notice recording statute and a grantor-grantee index system.

If the woman sues to eject the friend, will the woman be likely to succeed?

A No, because the friend took possession of the land before the woman did.

B No, because the neighbor’s title was superior to the woman’s title.

C Yes, because the friend had notice of the conveyance from the man to the woman.

D Yes, because the woman, unlike the friend, took title under a warranty deed.

A

(B) is correct. Under a notice recording system, a subsequent bona fide purchaser (“BFP”) prevails over a prior grantee who failed to record. A BFP is a purchaser who gives valuable consideration and has no notice of the prior grant. Notice includes actual, record, or inquiry notice. Also, under the shelter rule, a person who takes from a BFP will prevail against any interest that the transferor-BFP would have prevailed against. This is true even where the transferee had actual knowledge of the prior unrecorded interest. Here, the neighbor was a BFP. The woman had not recorded her deed and had not taken possession of the land; the neighbor gave value without any notice of the woman’s claim. The neighbor’s title was superior to the woman’s title because of the recording statute. When the neighbor sold the land to the friend, the friend was protected under the shelter rule despite having actual knowledge of the woman’s interest. Thus, the friend’s title is also superior to that of the woman.

(A) is incorrect. Possession isn’t required to establish title. The woman did not need to take possession before the friend to prevail; she needed to do something to put the first purchaser (the neighbor) on notice. If the woman had recorded her deed, the neighbor and the friend would not have had a valid claim to the property even if the woman had never taken possession of the property. Record notice would have put the neighbor on notice of her title, and he could not have been a BFP. The friend, taking from the neighbor, would not have been covered by the shelter rule. (If she did not record but took possession, that also would have put the neighbor on inquiry notice and he would not be a BFP.)

(C) is incorrect. Under the shelter rule, a person who takes from a BFP will prevail against any interest that the transferor-BFP would have prevailed against. This is true even where the transferee had actual knowledge of the prior unrecorded interest. Here, the neighbor was a BFP and the friend took from the neighbor. Thus, even though the friend had actual knowledge of the woman’s deed, he is still protected by the shelter rule and has superior title to the property.

(D) is incorrect. The type of deed does not affect who has title. A quitclaim deed has the same effect as a warranty deed in terms of conveying title. The differences among the types of deeds have to do with remedies available against the grantor if title turns out to be defective. A warranty deed gives the grantee contractual promises with respect to title, and the quitclaim deed gives no promises

How well did you know this?
1
Not at all
2
3
4
5
Perfectly
23
Q

Rule on purchase money mortgage.

A

The bank’s mortgage is a purchase-money mortgage, meaning that the funds the bank advanced were used to purchase the land. A purchase-money mortgage executed at the same time as the purchase of the real property encumbered takes precedence over any other claim or lien, including a previously filed judgment lien.

How well did you know this?
1
Not at all
2
3
4
5
Perfectly
24
Q

On the basis of scientific studies showing a causal relationship between the consumption of “red meat” (principally beef) and certain forms of cancer, a federal statute prohibits all commercial advertising of red meat products. The statute does not, however, restrict the sale of red meat products. Producers of red meat have challenged the statute as a violation of their free speech rights protected by the First Amendment.

Is the court likely to find the statute constitutional?

A No, because it does not serve a substantial government interest.

B No, because it is more extensive than necessary to serve the government interest in preventing certain cancers.

C Yes, because it does not affect speech protected by the First Amendment.

D Yes, because it serves a legitimate government interest in protecting public health.

A

(B) is correct. The regulation here involves commercial speech. Commercial speech is protected by the First Amendment, but the Court tests regulation of commercial speech under a special test. The Court first asks whether the speech is about a lawful activity and is truthful and not misleading. If these conditions are not satisfied, the speech has no protection. If they are satisfied, the regulation will be valid only if it (1) serves a substantial government interest, (2) directly advances that interest, and (3) is narrowly tailored to achieve that interest (that is, there is a reasonable fit between the means chosen and the ends sought). Here, the sale of red meat is allowed and so the producers are looking to advertise about a lawful activity. The regulation bans all commercial advertising. While the regulation serves a substantial government interest (that is, discouraging the consumption of a product linked to cancer), the regulation imposes a complete ban on advertisement. A complete ban will never be found to be narrowly tailored.

(A) is incorrect. The government will no doubt be found to have a substantial interest in preventing cancer.

(C) is incorrect. As indicated above, commercial speech is protected by the First Amendment.

(D) is incorrect. First, it is not enough merely that the interest served is legitimate - it must be substantial. And second, even if a legitimate interest were enough, that is only one prong of the test, and the regulation here would still fail because it is not narrowly tailored.

How well did you know this?
1
Not at all
2
3
4
5
Perfectly
25
Q

[comes up a lot]
A man who owned a business believed that one of his employees was stealing computer equipment from the business. He decided to break into the employee’s house one night, when he knew that the employee and her family would be away, to try to find and retrieve the equipment.

The man had brought a picklock to open the employee’s back door, but when he tried the door, he found that it was unlocked, so he entered. As the man was looking around the house, he heard sounds outside and became afraid. He left the house but was arrested by police on neighborhood patrol.

What is the man’s strongest defense to a burglary charge?

A The back door to the house was unlocked.

B The burglary was abandoned.

C The house was not occupied at the time of his entry.

D The man did not intend to commit a crime inside the house.

A

(D) is correct. Burglary is the breaking and entering of the dwelling of another at nighttime with the intent to commit a felony therein. Here, the man intended to retrieve the computer equipment that he thought the employee was stealing from his business. Since the man only wanted to take back property that was rightfully his, he did not have the intent to commit a crime. Thus, his strongest defense to burglary is that he did not intend to commit a crime inside the house. (A) is incorrect. The breaking necessary for burglary requires some use of force to gain entry, but minimal force is sufficient. Opening a closed but unlocked door constitutes a breaking. Here, the man opened the unlocked back door, so his actions are sufficient for burglary. (B) is incorrect. Burglary requires that the defendant have the intent to commit a felony at the time of entering the dwelling, but it is not necessary that the felony be completed or even attempted. Here, the man intended to retrieve computer equipment, but he got scared and left the house without taking anything. The fact that he did not go through with his plan does not affect his culpability for burglary. (C) is incorrect. A structure is a dwelling if it is used with regularity for sleeping purposes. The occupants do not have to be present during the burglary, and the occupants’ temporary absence does not deprive the structure of its character as a dwelling. Here, the employee and her family lived in the house and were away that night. Their temporary absence did not affect the house’s status as a dwelling.

How well did you know this?
1
Not at all
2
3
4
5
Perfectly
26
Q

Tell me about the parol evidence rule.

A

If the writing is only a partial integration, and not a complete embodiment of the parties’ intentions, under the parol evidence rule, it cannot be contradicted, but it may be supplemented by proving up consistent additional terms.

How well did you know this?
1
Not at all
2
3
4
5
Perfectly
27
Q

A toy collector had purchased 10 antique toys over the last several years and had had them restored by an expert in toy restoration. On June 1, the collector sent the 11th antique toy to the expert with a signed note that read: “Here is another toy for you to restore. As with all prior jobs, I will pay $500 for the work, but no more.” On June 4, after receipt of the collector’s June 1 note and the toy, the expert began restoring the toy. On June 6, the collector unexpectedly died. On June 7, unaware of the collector’s death, the expert sent the collector a note that stated that the restoration work had begun on June 4. The following day, the expert learned of the collector’s death.

Does a contract exist that binds the expert and the collector’s estate?

A Yes, because the expert sent the June 7 note before learning of the collector’s death.

B Yes, because the offer was accepted before the collector’s death.

C No, because the collector died before the expert sent the June 7 note.

D No, because the offer lapsed when the collector died.

A

(B) is correct. The offer was accepted before the collector’s death. The collector’s letter was an offer. It created a reasonable expectation in the expert that the collector was willing to enter into a contract on the basis of the offered terms. Unless an offer specifically provides that it may be accepted only through performance, it will be construed as an offer to enter into a bilateral contract and may be accepted either by a promise to perform or by the beginning of performance. Here, the expert began performance and, thus, accepted the offer when he began restoring the toy on June 4 - two days before the collector died on June 6th.

(A) is incorrect. The collector’s offer was accepted on June 4 when the expert began performance. Whether the expert knew of the collector’s death on June 7 is not relevant. Indeed, if the expert had not already accepted the offer before the collector’s death, the offer would have terminated on the collector’s death by operation of law, even if the expert did not know of the death.

(C) is incorrect. The June 7 note was not an acceptance. The acceptance had already occurred on June 4 when the expert began performance by beginning to restore the toy.

(D) is incorrect. If either of the parties to a proposed contract dies prior to acceptance, the offer is terminated. Here the collector’s offer had already been accepted at the time of the collector’s death. Thus, there was no lapse.

How well did you know this?
1
Not at all
2
3
4
5
Perfectly
28
Q

A husband and wife acquired land as common law joint tenants with right of survivorship. One year later, without his wife’s knowledge, the husband executed a will devising the land to his best friend. The husband subsequently died.

Is the wife now the sole owner of the land?

A No, because a joint tenant has the unilateral right to end a joint tenancy without the consent of the other joint tenant.

B No, because the wife’s interest in the husband’s undivided 50% ownership in the land adeemed.

C Yes, because of the doctrine of after-acquired title, or estoppel by deed.

D
Yes, because the devise to the friend did not sever the joint tenancy.

A

(D) is correct. Although as a general rule a joint tenant’s interest is freely alienable during his or her lifetime without the consent of the other joint tenant, that interest cannot be devised in a will. In this case, on the death of the husband, the wife’s interest in the joint tenancy immediately expanded and she became the sole owner of the land as the surviving joint tenant.

(A) is incorrect. As a general rule, a joint tenant’s interest is freely alienable during his or her lifetime without the consent of the other joint tenant. However, as stated above, a joint tenant’s interest cannot be devised in a will. In this case, on the death of the husband, the wife became the sole owner of the land as the surviving joint tenant.

(B) is incorrect. The doctrine of ademption applies only when an individual dies testate and attempts to devise land that the testator no longer owns. That doctrine is not applicable here because the wife became the sole owner of the land as the surviving joint tenant.

(C) is incorrect. The doctrine of after-acquired title, or estoppel by deed, applies when an individual attempts to convey title (usually by warranty deed) at a time when the individual does not have title to the land but later acquires title to the land. In this case, the husband had an interest in the land but did not have the power to devise it in a will, so the doctrine does not apply.

How well did you know this?
1
Not at all
2
3
4
5
Perfectly
29
Q

RAP : )
A woman owned land in fee simple absolute. The woman conveyed the land to a friend “for life,” and when the friend died the land was to go to the woman’s neighbor “and her heirs.”
The neighbor died and in her duly probated will devised her entire estate to a local charity. If she had died intestate, her daughter would have been her only heir.

One year after the neighbor died, her daughter executed a quitclaim deed conveying any interest she might have in the land to the woman’s friend.

The common law Rule Against Perpetuities is unmodified in the jurisdiction. There are no other applicable statutes.

Who has what interest in the land?

A The friend has a fee simple absolute, because his life estate merged with the remainder conveyed to him by the daughter.

B The friend has a life estate and the charity has a vested remainder, because the neighbor’s interest was devisable.

C The friend has a life estate and the daughter has a vested remainder, because the deed from the woman created an interest in the neighbor’s heirs.

D The friend has a life estate and the woman has a reversion, because the neighbor’s remainder was void under the Rule Against Perpetuities.

A

(B) is correct. The grant here gave a life estate to the friend and a remainder in fee simple to the neighbor. It is a remainder because on the friend’s death (the natural termination of the preceding estate), the neighbor has the right to possession. The remainder is vested because it was granted to an ascertainable person in being (the neighbor) and there were no conditions to prevent it from becoming possessory. A vested remainder is devisable by will; so when the neighbor devised her estate to the local charity, the charity took her vested remainder interest.

(A) is incorrect. Whenever the same person acquires all of the existing interests in land, present and future, a merger occurs, and that person then holds a fee simple absolute. Here, however, the daughter had nothing to convey to the friend. The woman conveyed a vested remainder in fee simple to the neighbor. That vested remainder is devisable, and the neighbor properly devised it by will to the local charity. The daughter, as an intestate heir, would have taken no interest in the property. A quitclaim deed conveys only the interest the grantor has, and in this case, that was nothing. Therefore, there was no merger. The daughter’s deed conveyed no interest to the friend, and the friend has only his life estate.

(C) is incorrect. The phrase in the grant “and her heirs” indicates that a fee simple interest was being granted; it does not give the heirs any rights in the property. Thus, as discussed, the daughter acquired no interest in the remainder.

(D) is incorrect. The Rule Against Perpetuities states that no interest in property is valid unless it must vest, if at all, within 21 years after a life in being at the creation of the interest. Here, the neighbor’s interest is already vested on creation; she has a vested remainder. Her estate is a vested remainder because it followed the natural termination of the preceding estate, and it was granted to an ascertainable person in being without any conditions to prevent it from becoming possessory. Because it is a vested remainder, it is valid under the Rule.

How well did you know this?
1
Not at all
2
3
4
5
Perfectly
30
Q

A man became ill while at work and decided to go home early. When he entered his bedroom, the man saw his wife engaged in sexual intercourse with a neighbor. The man grabbed a gun from a dresser drawer and shot and killed the neighbor. He was later charged and prosecuted.

In a jurisdiction that follows the common law for homicide offenses, which crimes should the court instruct the jury on?

Press Enter or Space to submit the answer

A Murder and involuntary manslaughter.

B Murder and voluntary manslaughter.

C Murder, voluntary manslaughter, and involuntary manslaughter.

D Voluntary manslaughter and involuntary manslaughter.

A

(B) is correct. The court should instruct the jury on murder and voluntary manslaughter. Murder is the unlawful killing of another human being with malice aforethought. Malice aforethought exists if the defendant has one of the following states of mind: (i) the intent to kill, (ii) the intent to inflict great bodily injury, (iii) reckless indifference to an unjustifiably high risk to human life, or (iv) the intent to commit a felony. Here, the man grabbed a gun and shot and killed the neighbor. A jury could find that he had the intent to kill, or at least the intent to inflict great bodily injury. The court should therefore instruct the jury on murder. Voluntary manslaughter is a killing that would otherwise be murder but is distinguishable from murder by the existence of adequate provocation. Provocation will reduce a killing to voluntary manslaughter if it meets four tests: (i) the provocation must have been one that would arouse sudden and intense passion in the mind of an ordinary person such as to cause him to lose his self-control; (ii) the defendant must have in fact been provoked; (iii) there must not have been a sufficient time between the provocation and the killing for the passions of a reasonable person to cool; and (iv) the defendant in fact did not cool off between the provocation and the killing. Adequate provocation is frequently recognized in the case of one spouse discovering the other in bed with another person. Here, the man’s actions meet the requirements for voluntary manslaughter. He caught his wife in bed with the neighbor, which would provoke an ordinary person and provoked the man himself. It appears that there were only a few seconds between the provocation and killing, which would be insufficient time for a reasonable person to cool, and nothing indicates that the man cooled off. Because the man’s actions fit the requirements for voluntary manslaughter, the jury should be instructed on that crime. If the jury were to find that this killing did not qualify as voluntary manslaughter, the man would be guilty of common law murder. Thus the jury should be instructed on both crimes.

(A) is incorrect. The killing does not meet the requirements of involuntary manslaughter. There are two types of involuntary manslaughter: (i) a death caused by criminal negligence and (ii) a killing caused by an unlawful act, either a misdemeanor or a felony not included in felony murder. Here, there is no indication that the man was negligent; his actions appear to be intentional. Also, the man did not cause the neighbor’s death while committing an unlawful act; he merely came home early from work. Because the killing does not fit into the definitions of involuntary manslaughter, the court should not instruct the jury on it.

(C) is incorrect. As discussed above, the killing does not meet the requirements for involuntary manslaughter, and the jury should not be instructed on it.

(D) is incorrect. As explained above, the court should instruct the jury on murder but not on involuntary manslaughter.

How well did you know this?
1
Not at all
2
3
4
5
Perfectly
31
Q

A patient domiciled in State A sued a surgeon domiciled in State B in a federal court in State A, alleging claims for malpractice. The surgeon moved to dismiss the action for lack of personal jurisdiction. The court denied the motion and set discovery cutoff and trial dates.

The surgeon has appealed the denial of the motion.

Should the appellate court hear the merits of the surgeon’s appeal?

A No, because the appellate court lacks jurisdiction over the appeal.

B No, because the district court’s decision on jurisdiction is final.

C Yes, because a contrary appellate decision could terminate the action.

D Yes, because the surgeon’s personal-jurisdiction challenge raises a constitutional question.

A

(A) is correct. With certain exceptions by rule or statute, only final judgments may be appealed. A final judgment is one that disposes of the whole case on its merits. Since the case was still pending after the denial of a motion to dismiss on the basis of a lack of personal jurisdiction, the judgment here was not a final judgment and thus was not appealable.

(B) is incorrect because an order determining personal jurisdiction may be appealed (and is not within the sole discretion of the district court), but the order must be “final” (that is, it must dispose of all claims by all parties).

(C) is an incorrect statement of law and the final order rule. The order must be “final” at the trial court level, not be potentially made final by an appellate court ruling.

(D) again is an incorrect statement of law. The final order rule may not be circumvented by the mere raising of a constitutional issue.

How well did you know this?
1
Not at all
2
3
4
5
Perfectly
32
Q

Ten years ago, a couple bought a building and moved into its second-floor apartment with their teenage daughter. The couple operated a shoe store on the first floor of the building for many years. When the couple purchased the building, the area was predominantly rural and was zoned for nonresidential use. The municipality’s zoning is cumulative.
Five years ago, the municipality rezoned the area to single-family residential use. The daughter was not aware of this change, since she was away at college.

Recently, the daughter inherited the building from her parents. The daughter immediately moved into the apartment and took over the operation of the shoe store on the first floor. The daughter has learned that a developer is planning to build a large residential community in the area surrounding her building.

The daughter has asked her lawyer for advice regarding her ability to continue operating the shoe store.

Should the lawyer advise the daughter that she can continue to operate her shoe store?

A No, because the nonconforming use of the building terminated when the daughter’s parents died.

B No, but the municipality must pay her reasonable compensation for her loss resulting from the change in zoning.

C Yes, because the shoe store is a nonconforming use.

D Yes, because the zoning is cumulative and the building is also used for single-family residential purposes.

A

(C) is correct. A cumulative zoning ordinance creates a hierarchy of uses of land, and land that is zoned for a particular use may be used for the stated purpose or for any higher use. A residential use is higher than a nonresidential use. Here, the building was in an area originally zoned for nonresidential use. The daughter and her parents used the property for a business and their residence. This was appropriate under the cumulative zoning ordinance as the family’s uses met or exceeded the zoned use. Later, the area was rezoned for single-family residential use, which is a higher use than the shoe store. However, a use that exists at the time of passage of a zoning ordinance and that does not conform cannot be eliminated at once. Generally, the nonconforming use may continue indefinitely, but any change in the use must comply with the zoning ordinance. Because the shoe store existed at the time of the rezoning, the daughter may continue to operate the shoe store as a nonconforming use.

(A) is incorrect. Generally, the nonconforming use may continue indefinitely. The key is the use of the property and not the ownership. The parents’ deaths do not affect the zoning status of the building. Thus, the daughter may live in the building and operate the shoe store.

(B) is incorrect. The zoning power is limited by the “no taking without just compensation” clause of the Fifth Amendment. Nevertheless, zoning is generally not a “taking” unless it amounts to a physical appropriation of the property or denies the owner of all economic use, which would not be the case here (the daughter could use the property as a single-family residence). Because the property is a nonconforming use, she will be allowed to continue that use.

(D) is incorrect. A cumulative zoning ordinance creates a hierarchy of uses of land, and land that is zoned for a particular use may be used for the stated purpose or for any higher use. A single-family home is a higher use than an apartment building or a commercial use such as a shoe store. Because the shoe store is a lower use than a single-family residence, it would not be allowed in this area. Nor would the daughter’s apartment above the store. The store and apartment are allowed only because they are a nonconforming use.

How well did you know this?
1
Not at all
2
3
4
5
Perfectly
33
Q

In a jurisdiction that follows the common law and has adopted the bilateral requirement for conspiracy, what crime has the man committed?

A Attempted burglary.

B Attempted larceny.

C Conspiracy.

D Solicitation.

A

D) is correct. Solicitation consists of inciting, advising, or inducing another to commit a crime with the specific intent that the person solicited commit the crime. Here, the man asked the pool cleaner to disarm the alarm and leave a side door unlocked so that he could enter the house and steal the coin collection, and he offered her part of the proceeds from selling the collection. This was clearly an inducement for the cleaner to commit a crime, and the man had the specific intent that the cleaner commit the crime. He has therefore committed solicitation.

C) is incorrect. Conspiracy consists of an agreement between two or more persons, an intent to enter into an agreement, and an intent to achieve the objective of the agreement. Under the bilateral approach, at least two guilty minds are required for a conspiracy. If one person in a two-party agreement is only feigning agreement, the other party cannot be convicted of conspiracy. Here, the cleaner outwardly agreed to the man’s plan, but she was only pretending and immediately contacted the police. Because she did not intend to enter into an agreement with the man and did not intend to help him steal the coin collection, she did not have a guilty mind. The man therefore cannot be convicted of conspiracy.

How well did you know this?
1
Not at all
2
3
4
5
Perfectly
34
Q

when two people take adverse possession together they take as what?

A

As TIC NOT JT

How well did you know this?
1
Not at all
2
3
4
5
Perfectly
35
Q

Is a traffic stop a custodial?

A

NO not for Miranda anyways.

How well did you know this?
1
Not at all
2
3
4
5
Perfectly
36
Q

A man was charged with first-degree murder. Two lawyers were appointed to represent him because the prosecution planned to seek the death penalty. On the first day of trial, the air-conditioning malfunctioned in the courtroom, so the judge directed that the selection of the jurors take place in his chambers. Because of the large number of potential jurors, the judge directed that only one lawyer for the prosecution and one for the defense participate in the jury selection process. The defendant remained in the courtroom during the questioning of the jurors. Once the jury was selected, the trial was postponed until the next day, when the air-conditioning was again working.

Did the court’s jury selection process violate the defendant’s federal constitutional rights?

A No, because a defendant’s confrontation rights are limited to witnesses rather than jurors.

B No, because there was good cause to conduct jury selection outside the defendant’s presence.

C Yes, because excluding the defendant undercut the presumption of innocence.

D Yes, because jury selection is a critical stage at which a defendant is entitled to be present.

A

(D) is correct. Jury selection is a critical stage of trial at which the defendant is entitled to be present.

(A) may be true, but it is also off point. The problem here is not a confrontation right, but rather the right to participate in one’s own case.

(B) is incorrect because it does not present a proper legal standard. There is no “good cause” which would permit a judge to violate a defendant’s constitutional guarantees regarding the right to a jury.

(C) is incorrect because nothing in the facts present an issue regarding the presumption of innocence.

How well did you know this?
1
Not at all
2
3
4
5
Perfectly
37
Q

A horse breeder offered to sell a colt to his neighbor and they agreed on a purchase price. The horse breeder subsequently received a letter from the neighbor thanking him for the sale and summarizing their agreement. The letter contained the neighbor’s alleged signature. When the horse breeder attempted to set up transfer of the colt, the neighbor denied that she agreed to purchase it. In a breach of contract action against the neighbor, the horse breeder offers into evidence the letter. The horse breeder testifies that he is familiar with the neighbor’s handwriting and recognizes the signature on the letter as being hers.

Assuming appropriate objection by the neighbor, who claims that she did not sign the letter, how should the trial court rule on the admissibility of the letter?

A Exclude the letter for lack of foundation because lay opinion testimony regarding handwriting identification is not admissible.

B Exclude the letter unless its authenticity is established by a preponderance of the evidence.

C Admit the letter as authentic and instruct the jury accordingly.

D Admit the letter but instruct the jury that it is up to them to decide whether the letter is authentic.

A

[where there is an issue of authenticity, the jury is the one that has to decide]

The court should admit the letter and instruct the jury that it is up to them to decide whether the letter is authentic. Before a writing may be received in evidence, it must be authenticated by proof showing that the writing is what the proponent claims it is. All that is necessary is proof sufficient to support a jury finding of genuineness. The authenticity of a document is a preliminary fact to be decided by the jury. Here, the horse breeder’s testimony that he is familiar with the neighbor’s handwriting and that he recognizes the signature on the letter to be that of the neighbor is sufficient to support a jury finding of genuineness. Thus, the letter should be admitted and authenticity should be left to the jury to decide.

(C) is wrong because, as noted above, where there is a dispute as to the authenticity of a document, the issue of authenticity is a fact determination for the jury, not the judge, to decide.

How well did you know this?
1
Not at all
2
3
4
5
Perfectly
38
Q

discriminating against aliens except for self-government?

A

Yes for things like teachers (PRIMARY and SECONDARY ONLY), police officers or probation officer.

How well did you know this?
1
Not at all
2
3
4
5
Perfectly
39
Q

(*) A homeowner, a citizen of State A, hired an electrician, a citizen of State B, to fix the wiring in her basement and hired a gas worker, also a citizen of State B, to install a new gas stove in her kitchen. Unfortunately, the home caught fire and burned down while they were both working on their separate jobs. The homeowner sued the gas worker for negligence in federal court in State A, seeking $100,000. The homeowner promptly served the gas worker, and the gas worker timely filed an answer with the court. One month after filing the answer, the gas worker moved to file and serve a third-party complaint against the electrician, alleging that the electrician was the sole cause of the accident.

Which of the following arguments is most likely to achieve the electrician’s goal of dismissal of the third-party complaint?

A The gas worker’s motion for leave to file a third-party complaint is untimely and thus should be denied as a matter of law.

B The court does not have subject matter jurisdiction over the third-party complaint because the electrician’s claim and the gas worker’s claim do not arise from a common nucleus of operative fact.

C The gas worker’s claim against the electrician is not a proper third-party claim.

D Dismissing the gas worker’s claim will not impede his ability to protect his rights in a separate action.

A

The electrician’s best argument is that the gas worker’s claim against the electrician is not a proper third-party claim. Under Rule 14, a defendant may assert a third-party claim against “a nonparty who is or may be liable to it for all or part of the claim against it.” In other words, a third-party claim must be a derivative claim; the third-party plaintiff must be seeking indemnification or contribution from the third-party defendant. Here, the gas worker’s claim is not that the electrician must indemnify him or that the electrician is a joint tortfeasor who may be jointly liable under principles of contribution. Rather, the gas worker is alleging that he (the gas worker) is not liable and that the electrician is. Because the claim is not derivative, it is not properly asserted as a third-party claim under Rule 14.

(A) is incorrect. A defendant may serve a third-party complaint as of right within 14 days of serving his original answer. Thereafter, he must make a motion to serve the complaint, and it is within the trial court’s discretion whether to grant or deny the motion. Here, it is unlikely that a court would deny a defendant’s motion to serve a third-party complaint at such an early stage of the proceeding.

How well did you know this?
1
Not at all
2
3
4
5
Perfectly
40
Q

A locksmith knew that his friend had been having marital troubles. The friend had told the locksmith that he suspected his wife was having an affair with his rival. One afternoon, the friend, visibly upset, asked to borrow some of the locksmith’s tools, telling him that he knew that his rival was going to meet up with his wife later that day. The locksmith gave his friend the tools, advising him not to do anything that he would regret later. The friend stated that it would be others who would have regrets. The friend went to his rival’s apartment and picked the door lock with the locksmith’s tools. He found his wife and rival in bed together. The friend stabbed his rival, seriously wounding him. A few minutes later the locksmith called the apartment to try to warn the rival that his friend might come over. After the friend was arrested, he agreed to plead guilty to aggravated battery and attempted voluntary manslaughter in exchange for testifying against the locksmith, who was charged as an accomplice to attempted murder.

Can the locksmith be convicted of that charge?
A Yes, because he recklessly disregarded a substantial risk to human life and was not provoked.

B Yes, because his failed attempt to neutralize his assistance did not prevent the crime from occurring and therefore did not constitute an adequate withdrawal.

C No, because he did not have the requisite intent to be liable as an accomplice.

D No, because an accomplice cannot be found guilty of a more serious offense than that for which the principal has been convicted.

A

The locksmith cannot be convicted as an accomplice because he did not have the requisite intent for attempted murder. To be convicted as an accomplice under the prevailing rule, a person must have given aid, counsel, or encouragement with the intent to aid or encourage the principal and the intent that the principal commit the substantive offense. Mere knowledge that a crime would result from the aid provided is generally insufficient for accomplice liability. Here, the locksmith did not provide the tools to the friend with the intent that he kill the rival. His knowledge that the friend might be intending harm to the rival is not sufficient to establish the intent to kill required for attempted murder.

(A) is incorrect because even if the locksmith’s conduct constituted reckless disregard of high risk to human life, that state of mind is not sufficient for attempted murder. Unlike murder, attempted murder is a specific intent crime and requires the intent to kill.

(B) is incorrect. Although the locksmith’s attempt to neutralize his assistance would not have been enough to raise the defense of withdrawal if he had incurred liability as an accomplice, here he did not have the requisite intent for accomplice liability.

(D) is an incorrect statement of law; the degree of liability of a principal is irrelevant to the potential liability of an accomplice. If the locksmith had had the intent to aid his friend in killing the rival, the fact that the friend could show adequate provocation to reduce his offense to attempted voluntary manslaughter would have no effect on the locksmith’s liability for attempted murder.

How well did you know this?
1
Not at all
2
3
4
5
Perfectly
41
Q

Based on recommendations of a state commission studying the effect of pornographic films on violent criminal activity, a state adopted legislation banning films intended for commercial distribution that appealed as a whole to the prurient interest in sex of the average person in the community, portrayed sex in a patently offensive way to citizens of the state, and which a reasonable person in the United States would find had no serious literary, artistic, political, or scientific value.

In ruling on a constitutional challenge to the legislation from a film distributor in the state who was convicted of distributing films in violation of the legislation, will the federal court likely find the legislation to be constitutional?

A Yes, because it uses a national “reasonable person” standard for determining the social value of the work.

B Yes, because it uses a statewide standard rather than a community standard for determining whether the material is patently offensive.

C No, because it uses a statewide standard rather than a national standard for determining whether the material is patently offensive.

D No, unless the court finds that the legislation is necessary to advance the state’s compelling interest in reducing violent criminal activity.

A

The court will likely find the legislation to be a constitutional regulation of obscenity. Obscenity, which is not protected speech under the First Amendment, is defined by the Supreme Court as a description or depiction of sexual conduct that, taken as a whole, by the average person, applying contemporary community standards, appeals to the prurient interest in sex, portrays sex in a patently offensive way, and—using a national reasonable person standard—does not have serious literary, artistic, political, or scientific value. Thus, the legislation here is constitutional because it uses a reasonable person standard, rather than a community standard, for determining the value of the work.

(B) is incorrect because while a statewide standard for determining whether the material is patently offensive is permissible, it is not mandatory. A state may use a “community standard” for making this determination.

(C) is incorrect because, again, a statewide standard for determining whether the material is patently offensive is permissible. Only the “social value” element of the obscenity test requires a national standard.

(D) is incorrect because the legislation is valid regardless of whether it is necessary to achieve the state’s compelling interest in reducing violent crime. Speech that falls within the definition of obscenity is unprotected speech; the government does not need a specific compelling interest to ban it.

How well did you know this?
1
Not at all
2
3
4
5
Perfectly
42
Q

What are requried pre trial disclosures?

A

The defendant must disclose the existence of the insurance policy under the Federal Rules, which expressly permit discovery of insurance agreements as an initial disclosure.

(1) the names, addresses, and phone numbers of individuals who contributed to the discovery, (2) a duplicate description of all related paperwork, compilation of all information pertaining to the invention, and publicly owned tangible objects, objects in custody, (3) a computation of damages, and (4) any related insurance coverage agreements.

How well did you know this?
1
Not at all
2
3
4
5
Perfectly
43
Q

To finance the purchase of a vineyard, a vintner borrowed $500,000 from a bank, secured by a mortgage on the vineyard. Due to a clerical error, the bank’s mortgage was not immediately recorded. Six months later, the vintner borrowed $10,000 from a creditor, also secured by a mortgage on the vineyard. The creditor immediately recorded its mortgage. The following week, the bank discovered its error and recorded its mortgage. Subsequently, the vintner defaulted on her payments to the bank. The bank instituted foreclosure proceedings but did not join the creditor in the action. A buyer purchased the property at the foreclosure sale.

A statute of the jurisdiction provides, “No conveyance or mortgage of real property shall be good against subsequent purchasers for value without notice unless the conveyance is recorded.”

If a court finds that the buyer took title subject to the creditor’s mortgage, what is the most likely reason?

A The creditor’s mortgage was senior to the bank’s purchase money mortgage.

B The bank did not join the creditor in the foreclosure action.

C The vintner has not defaulted on the creditor’s mortgage.

D The buyer assumed the creditor’s mortgage.

A

First of all here they treat creditors as bona fide purchasers
Also purchase money mortgage CAN be inferior to mortgage that became senior through a recording statutte.

f a court finds that the buyer took title subject to the creditor’s mortgage, it will be because the creditor’s mortgage was senior to the bank’s purchase money mortgage (“PMM”). A PMM is a mortgage typically given to a third-party lender, who is lending the funds to allow the buyer to purchase the property. A PMM, whether recorded or not, has priority over mortgages, liens, and other claims against the mortgagor that arise prior to the mortgagor’s acquisition of title. However, PMM priority is subject to being defeated by subsequent mortgages or liens by operation of the recording acts. Here, the creditor’s mortgage has priority over the bank’s PMM under the jurisdiction’s notice statute because the creditor had no notice of the bank’s interest at the time of the loan. Because foreclosure does not affect any interest senior to the mortgage being foreclosed, the buyer takes title subject to the creditor’s mortgage. (B) is incorrect because, although failure to join a necessary party results in the preservation of that party’s interest despite foreclosure and sale, the creditor’s mortgage was senior to the bank’s mortgage, and thus it was not a necessary party.

How well did you know this?
1
Not at all
2
3
4
5
Perfectly
44
Q

A plaintiff sued a defendant for defamation, asserting in her complaint that the defendant had called the plaintiff a thief in front of a number of business associates. The plaintiff calls two witnesses to the stand, both of whom testify that they heard the defendant refer to the plaintiff as a thief in front of the business associates. The plaintiff does not take the stand herself. The defendant pleads truth of the statement as an affirmative defense and calls a witness to the stand. The defense witness is prepared to testify that he was a co-worker of the plaintiff when the plaintiff supplemented her income by tending bar three nights a week. The witness will testify that he saw the plaintiff take a $20 bill from the tavern’s cash register and secrete the money in her pocket. The plaintiff’s attorney objects.

May the defense witness’s testimony be allowed?

Press Enter or Space to submit the answer

Correct
A Yes, as substantive evidence that the plaintiff is, in fact, a thief.

Incorrect
B Yes, because theft is a crime indicating dishonesty.

C No, because specific bad acts may not be used to show bad character.

D No, because the plaintiff never took the stand.

A

The defense witness’s testimony is admissible character evidence because the plaintiff’s character is directly in issue in the case. As a general rule, evidence of character to prove the conduct of a person in the litigated event is not admissible in a civil case. However, when proof of a person’s character, as a matter of substantive law, is an essential element of a claim or defense in a civil action, character evidence is admissible because it is the best method of proving the issue. Under the Federal Rules, any of the types of evidence-reputation, opinion, or specific acts-may be used. Here, character is an issue in the plaintiff’s defamation action because the defendant has pleaded as an affirmative defense that his statement claiming that the plaintiff is a thief is the truth. The defense witness’s testimony that he saw the plaintiff take the money from the cash register is relevant because it tends to show that the defendant spoke the truth. Hence, it should be allowed.

(B) is incorrect because the fact that the theft here could be considered a crime of dishonesty would be relevant only if the plaintiff’s credibility were being impeached, and only then if proof of an actual conviction were provided. Here, the testimony is admissible because it is being offered as substantive evidence of an aspect of the plaintiff’s character that is an essential element of a defense in the case.

How well did you know this?
1
Not at all
2
3
4
5
Perfectly
45
Q

(*)
A single man with a life insurance policy that pays his designated beneficiary $70,000 upon his death was killed in a car accident. His former girlfriend, a resident of State A, was named as beneficiary, but his mother, a resident of State B, also filed a claim for the life insurance proceeds. The insurance company, a State C corporation having its principal place of business in State B, filed an interpleader action in federal court to protect itself from potentially multiple and inconsistent claims.

May the insurance company bring the interpleader action in federal court?

A No, because the amount in controversy does not exceed $75,000, and there is no diversity between the insurance company and one of the claimants.

B No, because there is no federal court where the insurance company will be able to establish personal jurisdiction over both claimants.

C No, because the insurance company is not diverse from one of the claimants.

D Yes, because the amount in controversy is $500 or more, and both claimants are diverse from one another.

A

The insurance company can bring the interpleader action in federal court. The Federal Interpleader Act provides special rules for dealing with subject matter jurisdiction, personal jurisdiction, and venue. Under the act, interpleader can be brought if any two claimants are citizens of different states and the amount in controversy is $500 or more. Complete diversity is not required. Here, the mother is a State B resident, and the former girlfriend is a State A resident. Thus, both claimants are diverse from one another, and the $70,000 amount in controversy exceeds the $500 minimum.

(A) is wrong because it states the general diversity of citizenship subject matter jurisdiction requirements that would be applicable to Rule interpleader. Statutory interpleader has different requirements.

(B) is wrong because the Federal Interpleader Act permits the federal court to serve process anywhere in the United States. Therefore, as long as all the claimants reside or can be found anywhere in the country, the federal court can exercise personal jurisdiction over them.

(C) is wrong because the Federal Interpleader Statute does not require complete diversity. It only requires diversity between the two contending claimants, which exists in this case because the mother is a State B resident and the former girlfriend is a State A resident.

How well did you know this?
1
Not at all
2
3
4
5
Perfectly
46
Q

A state study indicated that an inordinately high percentage of homeless in the state were afflicted by alcoholism or addiction to illegal drugs. The legislature therefore decided to levy a special tax, with all proceeds marked for rehabilitative services for the homeless. However, the legislators determined that direct taxes on alcoholic beverages would be resented by the citizenry. Lobbyists from the state’s growing wine industry also objected to anything that would retard the industry’s development. There were no breweries or distilleries within the state. Thus, a tax was eventually passed requiring newspapers and magazines of general circulation published in the state to be taxed at a rate of 20% on all advertising space sold for beer or distilled spirits promotions.

For certain historical reasons, a high proportion of the advertising revenue of a particular small newspaper within the state came from beer and wine ads. The publisher of the small paper filed suit to have the tax declared unconstitutional. A major wholesale beer and liquor distributor located within the state and several out-of-state brewers and distillers who sold and advertised their products in the state also joined in the suit as plaintiffs.

If the tax is declared unconstitutional, what is the most likely reason?

A The tax burdens interstate commerce by exempting advertisements for the local wine industry from the tax, while the ads of out-of-state brewers and distillers are subject to the tax.

B The tax infringes on freedom of the press, which is guaranteed by the First and Fourteenth Amendments.

Incorrect
C The tax is unconstitutional because it is not properly apportioned.

D The tax violates the Equal Protection Clause of the Fourteenth Amendment, because it does not treat all alcoholic products equally.

A

The tax unconstitutionally burdens the freedom of the press. Press and broadcasting companies can be subject to general business taxes, but a tax applicable only to the press or based on the content of a publication will not be upheld absent a compelling justification. Mere need for revenue probably is not a sufficiently compelling interest. (A) is incorrect because there is no unconstitutional burden on interstate commerce here. The law treats all businesses subject to the tax (namely breweries and distilleries) equally, and so is not protecting local business against out-of-state competition. The fact that the law treats breweries and distilleries differently from wineries and that the state has no breweries or distilleries but does have wineries probably does not change this, because a court will probably find these to be distinct businesses for purposes of advertising. (C) is incorrect. When a sales tax is imposed on a sale taking place entirely within one state, there is no apportionment problem because the sale cannot be taxed by any other state (because no other state has a sufficient nexus).

How well did you know this?
1
Not at all
2
3
4
5
Perfectly
47
Q

A woman was late for an appointment with her doctor across town. Because of this, she was driving recklessly through traffic at a high speed and ran through a red light. There were a number of people crossing the street at the time, and the woman accidentally hit one of them. The person she hit was seriously injured and was rushed to the hospital, but recovered. The woman was arrested and charged with attempted murder.

Will the woman likely be convicted?

A No, because she did not intend to hit anyone with her car.

B No, because she had not gone far enough in her actions to constitute attempt.

C Yes, because a person is presumed to intend the natural and probable consequences of her act.

D Yes, because from her recklessness, the intent to inflict serious bodily harm will be presumed.

A

Although the woman may have been guilty of murder had the pedestrian been killed, it does not necessarily follow that she is guilty of attempted murder when she almost killed the pedestrian. Murder does not require the intent to kill; an awareness of an unjustifiably high risk to human life will suffice. Like all attempt crimes, attempted murder is a specific intent crime. Thus, the intent to kill is required. Because the woman did not intend to kill the pedestrian, she cannot be convicted of attempted murder. (B) is wrong because, although the woman can avoid guilt for attempted murder, she cannot do so for the reason given here. Attempt requires an act beyond mere preparation for the offense. If the woman had the required intent to kill, her act of running down the pedestrian would be sufficient for attempted murder.

(C) is wrong. It is often loosely said that one is presumed to intend the natural and probable consequences of her act. This is not to be taken literally. It means that if a particular result is a natural and probable consequence of what a defendant does, the fact finder may draw the inference from such circumstance that the defendant intended that result. Here, however, it is likely that the jury would infer that the woman, who wanted to keep a doctor’s appointment, never intended to kill a pedestrian.

(D) is wrong for the same reason. It is simply another phrasing of (C). But, as explained above, attempted murder requires an actual intent to kill, not a fictitious, imputed, or constructive one

How well did you know this?
1
Not at all
2
3
4
5
Perfectly
48
Q

Due to budget shortages and a critical need of funding to fight a war, Congress enacted a $25 tax on each person flying into an airport in the five most popular vacation destinations in the country, as determined by Congress. The tax was implemented, and officials in the five destinations were outraged, fearing that the number of vacationers to the taxed destinations would decrease due to the tax.

If the tax is challenged in federal court by an official with standing, is the most likely result that the tax will be held constitutional?

A No, because it makes it significantly more difficult for persons to travel between the states.

B No, because the tax unfairly discriminates against certain vacation destinations by taxing them and not taxing other, similar vacation destinations.

C Yes, because the tax is necessary to achieve a compelling government interest.

D Yes, because Congress has plenary power to impose taxes to raise revenue.

A

The destination tax will likely be held constitutional under Congress’s taxing power. Congress has the power to lay taxes under Article I, Section 8, and a tax measure will usually be upheld if it bears some reasonable relationship to revenue production or if Congress has the power to regulate the taxed activity. Despite the protest from the officials of the affected locations, the tax here does appear to be related to revenue production and so will be upheld. (C) is incorrect because it is based on the wrong standard-the compelling interest test does not apply here. (A) is incorrect because the extent of the right to travel is not clearly defined. The Supreme Court has established that the right to travel from state to state is a fundamental right that may be violated by state laws designed to deter persons from moving into a state; however, the Court has not specifically applied this rule to the federal government or to the type of tax legislation present here. The state cases involved treating old vs. new residents differently for purposes of voting or some government benefit, which may have violated the Privileges or Immunities Clause of the Fourteenth Amendment, or treating outsiders differently from residents, which may have violated the Equal Protection Clause of the Fourteenth Amendment. Neither clause is applicable to the federal government, so neither analysis is appropriate. (

How well did you know this?
1
Not at all
2
3
4
5
Perfectly
49
Q

(*)
The owner of a television agreed to sell it to a neighbor for $250. The neighbor made a down payment of $70, took possession of the television and agreed to pay the outstanding balance in nine equal $20 installments, beginning on June 5, with subsequent installments due on the fifth of each month until the balance was paid in full.

The neighbor’s friend owed her $200. On May 20, the neighbor and her friend entered into an oral agreement whereby the friend agreed to make the nine $20 installment payments to the seller in exchange for the neighbor’s promise to forgive the friend’s $200 debt. On June 7, the seller called the neighbor to ask her where his first $20 installment payment was, and she told him at that time of her agreement with her friend. The friend has made none of the installment payments.

If the seller files suit against the friend demanding payment, who will prevail?

A The seller, because he was a third-party beneficiary of the agreement between the neighbor and her friend.

B The seller, because he is an assignee of the neighbor’s rights against her friend.

C The friend, because there was no consideration for her promise to the neighbor.

D The friend, because the surety provision of the Statute of Frauds prevents the seller from enforcing the friend’s promise.

A

The neighbor has delegated her duties under the agreement with the seller to her friend, and the friend has agreed to assume the duties by agreeing to make the installment payments to the seller. Where a delegate’s promise to perform the delegated duty is supported by consideration, there results a third-party beneficiary situation, so that the nondelegating party to the contract can compel performance or bring suit for nonperformance. The friend’s promise to make the payments to the seller, totaling $180, was given in exchange for the neighbor’s promise to forgive the $200 debt owed by the friend to her. The neighbor thus relinquished her right to take action against her friend for the full amount owed, thereby incurring legal detriment. Consequently, the promise of the friend was supported by consideration, and a situation arose in which the seller became a third-party beneficiary of the agreement between the neighbor and her friend, and able to enforce performance of the friend’s promise to pay.

(C) is incorrect because, as explained above, the friend’s promise to the neighbor was supported by consideration.

How well did you know this?
1
Not at all
2
3
4
5
Perfectly
50
Q

What are the elements of negligent misrepresentation?

A

negligent misrepresentation is not supported by these facts. Negligent misrepresentation requires (i) a misrepresentation made by defendant in a business or professional capacity, (ii) breach of duty toward that particular plaintiff, (iii) causation, (iv) justifiable reliance, and (v) damages. Here, even though the investor was involved in a business transaction, the attorney was not

51
Q

Elements of “equitable mortgage”

A

If a deed is given for security purposes rather than as an outright transfer of the property, it will be treated as an “equitable” mortgage and the creditor will be required to foreclose it by judicial action like any other mortgage. In determining whether an absolute deed is really a mortgage, the court considers the following factors: (i) the existence of a debt or promise of payment by the deed’s grantor; (ii) the grantee’s promise to return the land if the debt is paid; (iii) the fact that the amount advanced to the grantor/debtor was much lower than the value of the property; (iv) the degree of the grantor’s financial distress; and (v) the parties’ prior negotiations.

52
Q

Tell me about the penitent - clergy priviledge.

A

Does NOT apply wherethe statement was made in confidence but in a social situation.

53
Q

A mother and father instructed their son who just turned age 14 to report to a community woodworking shop instead of school. A state law requires all children to attend school until the age of 16, and the woodshop does not qualify as a school under state law. Because the parents did not report their son’s absence, a truant officer visited the family and warned them that parents who willfully refuse to comply with the mandatory attendance law are subject to a $500 fine and up to 30 days in jail for each day of noncompliance. The parents listened, but informed the officer that they could not comply with the state law because of their religious views, under which woodworking is an essential spiritual pursuit. The following day, the 14-year-old again went to work in the community woodshop instead of to school. His parents were then arrested and charged with violating the state mandatory school attendance law.

At the parents’ criminal trial, which of the following may the court constitutionally consider in determining whether First Amendment protection applies to the parents’ views?

A Whether the parents’ belief system is related to an organized group that gathers regularly to express or celebrate the belief system.

B Whether the parents’ beliefs are derived from a traditional religion.

C Whether the parents’ belief system includes recognition of a supreme being.

D Whether the parents’ belief system occupies a place in their lives similar to that occupied by orthodox religious beliefs.

A

Although the validity of religious beliefs cannot be questioned, the role a belief system plays in a person’s life can be examined to determine whether the beliefs are indeed religious. The Free Exercise Clause of the First Amendment, applicable to the states through the Fourteenth Amendment, prohibits punishing people for their religious beliefs. When a person claims that he is being punished for his religious beliefs, the court may consider whether the person challenging the law sincerely holds those beliefs. Thus, the court may consider whether the parents’ beliefs play a role in their lives similar to that of orthodox religious beliefs. (A) is incorrect because religious beliefs do not have to be related to an organized group to be considered religious. (B) is incorrect because the Free Exercise Clause protects all sincerely held religious beliefs, regardless of whether a specific religion is deemed to be “established” or “traditional.” (C) is incorrect because religious beliefs need not involve belief in a supreme being to qualify for constitutional protection. An asserted religious belief must occupy a place in the believer’s life parallel to that occupied by orthodox religious beliefs. Even an internally derived belief is entitled to protection.

54
Q

Requirements for liquidated damages

A

The parties to a contract may stipulate what damages are to be paid in the event of a breach if (i) damages are difficult to ascertain at the time the contract is formed, and (ii) the amount agreed on is a reasonable forecast of compensatory damages in the case of a breach. These conditions have been met here.

55
Q

A minor entered a liquor store and asked the cashier to let him purchase a case of beer for a party even though he was underage. The cashier agreed as long as he paid double the retail price for the beer as compensation for the risks of the sale. As the cashier started to ring up the transaction, an undercover officer who overheard them intervened and arrested them. The state criminal code provides that it is a felony to knowingly provide alcohol to any person under the age of 21.

If the state follows the unilateral theory of conspiracy, of what crimes can the minor and cashier be convicted?

A The minor can be convicted of solicitation and the cashier can be convicted of conspiracy to violate the statute.

B Both the minor and the cashier can be convicted of conspiracy to violate the statute.

C The minor cannot be convicted of either solicitation or conspiracy, but the cashier can be convicted of conspiracy to violate the statute.

D The minor cannot be convicted of either solicitation or conspiracy, and the cashier cannot be convicted of conspiracy.

A

The cashier can be convicted of conspiracy in a unilateral jurisdiction even though the minor can be convicted of neither solicitation nor conspiracy. Under the unilateral approach adopted by the Model Penal Code, the crime of conspiracy requires that only one party have genuine criminal intent, and may be shown by proof that the defendant agreed with another to commit a crime, even if the other person does not share the commitment. Thus, the fact that no other party to the conspiracy could be found guilty does not prevent the defendant from being convicted of conspiracy. Here, the fact that the minor is a member of the class that the statute was designed to protect prevents him from being found guilty of conspiracy.

56
Q

(*)
An investor owned a 100-acre parcel that contained several natural asphalt lakes. A construction company was erecting highways for the state in the vicinity of the investor’s land and needed a supply of asphalt. The investor executed a document that, in return for a payment of $1 per barrel, gave the company the right to enter on the land and take asphalt in whatever quantities the company desired. The investor reserved the right to remove asphalt herself and to grant this right to others. Last year, the state commenced an action in eminent domain to take the investor’s land for a public park.

Is the construction company entitled to compensation?
A No, because the nonexclusive nature of the company’s right makes it a license, which is not an interest in property.

B No, because a nonexclusive profit, although an interest in property, has no value separate and apart from the land itself.

C Yes, because the company has a nonexclusive profit, which is a property right for which it is entitled to compensation.

D Yes, because the company has a license coupled with an interest, which is a property right for which it is entitled to compensation.

A

The construction company is entitled to compensation because it has a property right to enter and remove minerals.

Like an easement, a profit is a nonpossessory interest in land. The holder of the profit is entitled to enter on the servient tenement and take the soil or the substance of the soil (e.g., minerals, timber, oil, or game).

When an owner grants the sole right to take a resource from her land, the grantee takes an exclusive profit and is solely entitled to the resources, even to the exclusion of the owner of the servient estate.

By contrast, when a profit is nonexclusive, the owner of the servient estate may grant similar rights to others or take the resources herself. Although here the profit is nonexclusive, it is nevertheless an interest in property for which the company is entitled to compensation in any condemnation proceeding.

(A) is incorrect because a license is merely revocable permission to enter on another’s land. Unlike a profit, a license is not an interest in land; it is merely a privilege, ordinarily terminable at the will of the licensor. (B) is incorrect because a profit is the right to take something from another person’s land; it has a value apart from the land itself and is alienable. (D) is incorrect because a license coupled with an interest has the effect of making the license irrevocable, but it does not convert the license into an interest in land for which compensation is required.

57
Q

Requirements for judicial notice

A

The Federal Rules conform to the existing state rules governing judicial notice. Federal Rule 201(b) defines a fact that may be noticed as “one not subject to reasonable dispute in that it is either (i) generally known within the territorial jurisdiction of the trial court, or (ii) capable of accurate and ready determination by resort to sources whose accuracy cannot reasonably be questioned.” To be considered generally known within the community, the fact must be something that well-informed people generally know and accept. Although usually facts of common knowledge are known everywhere, it is sufficient for judicial notice if they are known in the community where the court is sitting.

58
Q

A publisher entered into a contract with a paper manufacturer who used very fine materials, whereby the publisher was given the right to purchase all paper refined by the paper manufacturer for the next five years at a price set at 95% of the domestic market price at the time of delivery. The publisher agreed to purchase no less than 1,000 pounds of paper a week. At the time this contract was signed, the publisher gave written notice to the paper manufacturer that it intended to buy all paper produced by the paper manufacturer until further notice. The paper manufacturer then sold its business to a lumber-processing company.

What is the effect of this sale on the paper manufacturer’s obligation to the publisher?

A The sale discharges the paper manufacturer’s obligation to the publisher because there has been a full performance.

B The paper manufacturer is liable for damages if the lumber processing plant fails to deliver paper to the publisher.

C The paper manufacturer is excused from further performance because it no longer has a factory to produce paper.

D The paper manufacturer breached its contract with the publisher.

A

The paper manufacturer is liable for damages if the lumber processing plant fails to deliver paper to the publisher. Because delivery of paper is not personal in nature, that duty can be delegated. The quantity will be measured by the paper manufacturer’s original output. However, when a duty is delegated to a delegate, the delegator remains liable should the delegate fail to perform. (A) is incorrect because the contract was for five years, and five years have not yet elapsed. (C) is wrong because a delegator remains liable. (C) would be correct only if the paper manufacturer went out of business without delegating its duties to another by selling the other the business, not the case here. (D) is wrong because, as indicated above, such a delegation is proper

59
Q

A man on parole after being convicted of possession of cocaine was suspected of selling cocaine out of his home. His parole officer came to his house and rang the bell. As soon as the man opened the door to see who was there, the officer entered the home, despite the man’s protests. After searching the home, the parole officer discovered several bags of marijuana in a drawer. The man was arrested and charged with possession of marijuana with intent to sell. A statute in the jurisdiction in which the search took place provides that, as a condition of parole, a parolee is on notice that his parole officer may conduct a search of the parolee’s person or home, without probable cause, at any time of the day or night. The man moved to have evidence of the marijuana suppressed by the court, claiming that the state statute that authorized the search was unconstitutional under the Fourth Amendment prohibition of unreasonable searches and seizures.

Will he prevail?

A Yes, unless probable cause was established by the officer’s tip in conjunction with other circumstances.

B Yes, because a search warrant was not obtained and no exception to the warrant requirement applies.

C No, because the man had a diminished expectation of privacy and the government has a heightened need to search parolees’ homes.

D No, because the search was incident to a lawful arrest.

A

The man will not prevail in his motion to suppress. To be reasonable under the Fourth Amendment, most searches must be pursuant to a warrant. However, several types of inspections and searches do not require a warrant or even probable cause. The Supreme Court has held that the Fourth Amendment is not violated by a statute authorizing warrantless searches of a parolee’s home-even absent probable cause-if a statute provides for such searches. The Court reasoned that in such circumstances, the parolee has a diminished expectation of privacy and the government has a heightened need for searching parolees; thus the search is reasonable in a constitutional sense. (A) is incorrect because while probable cause may be based on this type of tip under the “totality of the circumstances” test, probable cause is not necessary to establish the validity of the search based on the above discussion. (B) is incorrect because a search warrant is not required for a search of a parolee’s home that otherwise complies with procedures. (D) is incorrect because this was not a search incident to a constitutionally valid arrest. There was no basis for an arrest until after the search occurred; if the search were not otherwise independently valid, the fact that the man was arrested after the search revealed the drugs would not make the search valid.

60
Q

When are judgements enforceable?

A

Absent a court order, no execution on judgments is allowed for 30 days after entry except for injunctions or receiverships, which are not held up unless otherwise ordered by a court.

61
Q

A landowner validly conveyed a parcel of land to a veterinarian “for so long as the property is used as a veterinary practice, but if the property is used for any other purpose, it is to go to the American Cancer Society.” Two years later, the landowner died, validly devising all of his property to his friend. The landowner’s only heir is his daughter. Although this jurisdiction is a common law jurisdiction with respect to all real property considerations, the state’s probate laws provide that future interests or estates in real property may be passed by will or descent in the same manner as present or possessory interests.

Last month, the veterinarian approached the daughter and asked her to join with him to sell the parcel of land, which he had been using as an animal shelter, in fee simple absolute to a developer. The veterinarian and the daughter entered into a contract of sale with the developer. However, after consultation with an attorney, the veterinarian decided against the sale. The developer sued the daughter and the veterinarian for specific performance.

Will the requested relief likely be granted?

A No, because the American Cancer Society did not join in the contract of sale.

B No, because the friend did not join in the contract of sale.

C Yes, because the veterinarian had the power to sell his interest.

D Yes, because together, the daughter’s and the veterinarian’s interests would merge and they would have a fee simple estate.

A

The requested relief will be denied because the friend did not join in the contract of sale. The conveyance purported to create a fee simple determinable in the veterinarian subject to an executory interest in the American Cancer Society. A fee simple determinable subject to an executory interest is an estate that, on the happening of a stated event, is automatically divested in favor of a third person, who holds the executory interest. However, the executory interest in the American Cancer Society is void under the Rule Against Perpetuities because it might vest beyond lives in being plus 21 years. The charity-to-charity exception to the Rule does not apply because the veterinarian is not a charitable organization. Because any interest that violates the Rule is void and stricken from the instrument, what is left is a fee simple determinable in the veterinarian and a possibility of reverter in the landowner. On the landowner’s death, the possibility of reverter passed to the friend. (A) is incorrect because the American Cancer Society does not have any interest in the land. (C) is incorrect because, although the veterinarian had the power to sell his interest, he did not own a fee simple absolute. (D) is incorrect because the daughter does not have any interest in the land. Even if she did, the merger doctrine-which provides that whenever the same person acquires all of the existing interests in land, present and future, a merger occurs and that person holds a fee simple absolute-would not apply because the friend also has an interest in the land.

62
Q

A state statute provides: “Any merchant desiring to sell within this state any product or goods manufactured outside of the United States must (i) obtain a special license from the state for $50 and (ii) clearly mark the goods as to specify their country of origin.” The statute makes it a misdemeanor for any merchant to willfully sell goods without complying with these statutory requirements.

Which of the following statements is correct regarding the constitutionality of the statute?
A The portion of the statute requiring the license fee can be sustained on the ground that reasonable inspection fees are proper; but the balance of the statute is invalid.

B The portion of the statute requiring that the goods be labeled as to country of origin can be sustained because it only requires disclosure; but the balance of the statute is invalid.

C The statute is constitutionally valid as long as the burden on foreign commerce is minimal and is justified by legitimate state interests.

D The statute is unconstitutional in its entirety.

A

The statute is an unconstitutional violation of the Commerce Clause. Regulation of foreign commerce is exclusively a federal power because of the need for the federal government to speak with one voice when regulating commercial relations with foreign governments. The existence of legitimate state interests underlying state legislation will not justify state regulation of foreign commerce. The state statute, in imposing requirements for a license costing $50 and for a clear marking of goods as being from a foreign country, clearly is an attempt by the state to restrict or even eliminate the flow of such goods in foreign commerce. Thus, the statute is unconstitutional.

(A) is incorrect because even if the $50 fee represents a reasonable inspection fee, the fee would still constitute an interference with foreign commerce. In addition, the facts do not indicate that the license fee has anything to do with inspection, or that the amount of the fee bears any relation to legitimate inspection purposes.

(B) is incorrect because the labeling requirement imposes a burden on goods that flow in the stream of foreign commerce. Although this burden may be relatively small, it is still impermissible in light of the exclusive power held in this area by the federal government.

63
Q

do jury verdicts have to be unanimous in federal courts?

A

In federal civil cases, a verdict must be unanimous unless the parties agree to the contrary

64
Q

Wen is a witness treated as not being competent?

A

All witnesses are competent unless physically or mentally impaired in some fashion not applicable here, or unless they are too young to understand the oath and the need to testify truthfully.

65
Q

Dismissal with prejudice?

A

Under Federal Rule of Civil Procedure 41, dismissals for lack of prosecution are with prejudice and operate as an adjudication on the merits unless the court designates otherwise.

66
Q

A landowner executed a will, devising a parcel of land “to my sister for life, then to my brother for life, then to my nieces and nephews.” When the landowner died, he was survived by the sister and the brother’s son. The sister and the brother’s son contracted to sell the land to a buyer for $225,000. At the time set for closing, the sister and the brother’s son tendered a quitclaim deed to the buyer, who refused to complete the sale. The sister and the brother’s son bring suit against the buyer for specific performance. The jurisdiction in which the land is located does not follow the Doctrine of Worthier Title.

Will specific performance likely be granted?

A Yes, because a quitclaim deed conveys whatever interest the grantors have in the property.

B Yes, because the interests involved are freely alienable.

C No, because the jurisdiction does not follow the Doctrine of Worthier Title.

D No, because title is unmarketable.

A

Short answer: title was unmarketable beacuse the future interest could apply to nephews that are yet to be born before sister’s death.

Rule: Title may be unmarketable where the owners of the present and future interests attempt to convey a fee simple absolute title if the future interests are held by persons who are unborn or unascertainable

The sister and the brother’s son will not prevail in a suit against the buyer for specific performance because title is unmarketable. Marketable title is title reasonably free from doubt, i.e., title that a reasonably prudent buyer would be willing to accept. Title may be unmarketable where the owners of the present and future interests attempt to convey a fee simple absolute title if the future interests are held by persons who are unborn or unascertainable. Here, the sister has a life estate, and the brother’s son has a vested remainder subject to open because there may be other nieces and nephews born during the sister’s life who become entitled to share in the remainder. Life estates and vested remainders are freely transferable, which means that the sister and the brother’s son together can transfer the land to a purchaser, but the title is not marketable. It may turn out that the sister has a child, who is entitled to share in the remainder, but who did not join in the conveyance to the buyer. Because the child would not be bound by the conveyance, he would own an interest in the land. Thus, although the interests are considered alienable, the sister and the brother’s son cannot convey good title because there are outstanding interests in the unborn nieces and nephews.

67
Q

What is the Doctrine of WOrthier title?

A

In the common law of England, the doctrine of worthier title was a legal doctrine that preferred taking title to real estate by descent over taking title by devise or by purchase. It essentially provides that a remainder cannot be created in the grantor’s heirs, at least not by those words

68
Q

What is the Dead Man act?

A

A dead man’s statute, also known as a dead man act or dead man’s rule, is a statute designed to prevent perjury in a civil case by prohibiting a witness who is an interested party from testifying about communications or transactions with a deceased person against the decedent unless there is a waiver.

69
Q

A shoplifter set a small fire in a store’s trash basket. The shoplifter knew that the store’s automatic sprinkler system would promptly douse the fire, but his purpose was merely to create a distraction so he could walk out with an expensive watch, which would constitute felony theft. The shoplifter was stopped after he set the fire but before he could take any merchandise. No serious damage was done to the store by the fire, but the flame charred the wall next to the trash basket and blistered the paint on the surrounding area. A statute in the jurisdiction extends the crime of arson to buildings other than dwellings.

If the shoplifter is tried for the crime of arson, should the court find him guilty?

A No, because the shoplifter did not intend to burn the building.

B No, because the shoplifter’s act was sufficient only for attempted arson.

C Yes, because the shoplifter’s conduct demonstrated the requisite state of mind for the crime.

D Yes, because the shoplifter started the fire during the act of perpetrating another felony.

A

Short answer: arson is not speicifc intent you only need to intend to create a fire hazard not burn down the whole thing.

The shoplifter’s conduct satisfies the mens rea of “malice” required for arson. At common law, arson was defined as the malicious burning of the dwelling of another. The mens rea required for arson is malice, which is broader than the intent required for specific intent crimes and has nothing to do with ill will or evil motive. The defendant need not have intended to burn down the building; it is sufficient if he intended a burning that creates an obvious fire hazard to the building. Here, the shoplifter knew that the store had an automatic sprinkler system, and he was not motivated by an intent to burn down the building. Nevertheless, he intended to start a fire with reckless disregard of a high risk that it would cause damage to the building. The risk or hazard is not that the building will burn down, merely that damage to the structure from a burning will occur. The “burning” required for arson means a “charring” of the combustible material; mere “scorching” is insufficient. Here, the wall next to the trash basket was charred. This satisfies the “burning” requirement. The common law requirement that the structure be a dwelling has been broadened by the statute in this question to include other buildings. Thus, the shoplifter’s conduct satisfies all of the elements of the crime of arson.

(A) is incorrect because the mens rea for arson can be satisfied even in the absence of a specific intent to burn the building. He acted with the requisite intent-malice-by intentionally starting a fire that created a high risk that a burning of the structure would occur.

70
Q

Grounds to set aside an entry of default?

A credit card company filed a civil action against a consumer in federal district court seeking to recover the unpaid balance on the consumer’s account. The credit card company properly served process on the consumer, but the consumer failed to file or serve a timely answer to the complaint. The clerk of court on motion of the credit card company made an entry of default, but default judgment has not been entered.

On which grounds may the court set aside the entry of default?

A For the same limited grounds for which any court judgment may be set aside.

B Only if the court finds that the consumer could not have filed and served a timely answer despite using reasonable diligence.

C If the consumer demonstrates that there was good cause for his failure to file and serve a timely answer and that he has a viable defense.

D On any grounds that the court, in its discretion, finds just.

A

An entry of default may be set aside for “good cause shown.” Although not specifically required by the Federal Rules, a majority of courts also will require some showing of a meritorious defense.

(A) is incorrect because the grounds are not limited to the grounds required for setting aside regular court judgments.

(B) is incorrect because that answer is also too limiting on the concept of “good cause shown.” For example, a default might result from an honest mistake of the attorney, but the entry of default may nonetheless be set aside if the attorney acts promptly to correct the mistake.

(D) is incorrect because a majority of courts will also require a showing of a meritorious defense.

71
Q

A developer owned a large urban property, which she subdivided into 10 lots. The developer conveyed Lot 1 to an architect by a deed that contained a restriction banning commercial use of the property. The developer subsequently conveyed Lots 2 through 7 to six separate purchasers. Each of the deeds to these purchasers also contained the restriction on commercial use. The architect left Lot 1 undeveloped, but the purchasers of Lots 2 through 7 all used their lots for commercial purposes. The developer subsequently conveyed Lot 8 to a florist. The florist’s deed contained the restriction banning commercial use of the lot, but he decided that he wished to use Lot 8 commercially. The developer retains ownership of Lots 9 and 10. The florist wants to bring suit to establish his rights to use Lot 8 for commercial purposes.

Which of the following best describes the parties the florist should join in his lawsuit?

A The developer only.

B The developer and the architect only.

C The other commercial users only.

D All landowners in the subdivision.

A

Short answer: florist should joint everyone in the scheme because they could all be potential plaintiffs enforcing the equitable servitude (does not matter if they would likely not succeed).

The florist should join all of the landowners in the subdivision in a suit to terminate the servitude on the grounds of abandonment.

If a covenant in a subdivision deed is silent as to who holds its benefit, any neighbor in the subdivision will be entitled to enforce the covenant if a general scheme or plan is found to have existed at the time she purchased her lot.

In addition, a prior purchaser can enforce a restriction in a subsequent deed from a common grantor under either a third-party beneficiary theory or an implied reciprocal servitude theory.
Under the implied reciprocal servitude theory, an implied reciprocal servitude attaches to the grantor’s retained land at the moment she deeds a lot with the restriction.

Thus, all of the other landowners in the subdivision could potentially enforce the covenant as an equitable servitude against the florist. All parties would probably fail in an attempt to enforce the servitude, but the florist should join them now to avoid multiple litigation.

(Note that had the other landowners tried to enforce the equitable servitude against the florist, they would all have been subject to the equitable defense of acquiescence, which provides that if a benefited party acquiesces in a violation of the servitude by one burdened party, he may be deemed to have abandoned the servitude as to other burdened parties. In addition, the other commercial users are subject to the defense of unclean hands. It is important to remember that these are defenses and do not terminate the servitude; therefore, it would be best for the florist to join all possible complainants in a suit to have the servitude declared extinguished.)

72
Q

A plaintiff sues her employer for sexual harassment. During the trial, the plaintiff attempts to introduce into evidence company records that include written complaints from other employees alleging that they too were sexually harassed by the employer. The defense objects to the admission of the records on the basis of hearsay.

Should the objection be sustained?

A Yes, because the records are hearsay not within any recognized exception.

B No, because the records qualify under the business records exception.

C No, because the records qualify as a statement against interest.

D No, because the records are not hearsay.

A

The court should sustain the objection because the records are hearsay not within any recognized exception. Hearsay is a statement, other than one made by the declarant while testifying, offered into evidence to prove the truth of the matter asserted. Here, the records are being offered to prove that the employer sexually harassed other employees, to support the plaintiff’s contention that the employer sexually harassed her. Because the statements are offered to prove the truth of the matter asserted, they are hearsay, and because there is no recognized exception that would allow the records to be admitted, they must be excluded. Therefore, (D) is incorrect.

(B) is incorrect. The business records exception applies to records or writings made in the course of a regularly conducted business activity by one who was under a duty to do so. Here, because the employees were not under a business duty to file their claims, the business records exception does not apply to their statements.

73
Q

Can you take judicial notice of foreign law?

A

NO, not covered by Federal Law Rule on judicial notice.

74
Q

Use of car checkpoints?

A

Even absent that suspicion, police may set up roadblocks to stop cars if (i) the cars are stopped on the basis of some neutral, articulable standard, and (ii) the stops are designed to serve a purpose closely related to a particular problem arising from automobiles and their mobility. [See Indianapolis v. Edmund (2000)] The use of a checkpoint to detect evidence of ordinary criminal wrongdoing unrelated to use of cars or highway safety is unconstitutional.

75
Q

Thirty years ago, a power company constructed a power dam on a river. At the time the dam was constructed, the power company solicited and received express easements from all of the landowners in the river valley, including a farmer. The power company paid fair value for the easements, which would allow the company to release water from the dam at certain times of the year, resulting in flooding of the land in the river valley.

In the 30 years since the dam was constructed, the farmer’s property has never been flooded, and the farmer has been using his land in the same way as he did 30 years ago. Now, however, the power company wants to substantially increase power production from the dam. All landowners in the valley were notified by the company that henceforth all 200,000 acres (including the farmer’s 200 acres) would be flooded in accordance with the company’s rights under the easement. The farmer reviewed the easement for his property and discovered that it lacked the requisite grantor’s acknowledgment and thus was improperly recorded. The state’s adverse possession statute requires hostile occupation for a period of 20 years.

May the power company properly flood the farmer’s land under the terms of the easement?

A Yes, because such flooding is within the terms of the easement.

B No, because the state’s adverse possession statute requires hostile occupation for a period of only 20 years.

C No, because the company has failed to exercise its rights under the easement for 30 years, and the easement has lapsed.

D No, because the easement was not properly acknowledged and recorded.

A

The power company received an express easement entitling it to flood the farmer’s property.

To create an easement by express grant, there must be a writing signed by the grantor. If validly created, an easement is presumed to be of perpetual duration. No facts suggest that this easement was impliedly created or prematurely terminated.

(B) is incorrect because the requirements for extinguishing an easement by adverse use for the prescriptive period have not been fulfilled. To extinguish an easement by prescription, the owner of the servient tenement must so interfere with the easement as to create a cause of action in favor of the easement holder. The interference must be open, notorious, continuous, and nonpermissive for the prescriptive period. The farmer has done nothing (such as using his land in a different manner) that would indicate an interference with the power company’s easement so as to give rise to a cause of action in favor of the company

D) is incorrect because improper recordation does not affect the rights of the original parties to the transaction. To be properly recorded, an instrument generally must be acknowledged before a notary public. Although an unacknowledged instrument does not impart constructive notice to subsequent purchasers, it has absolutely no effect on the validity of the easement as between the original parties.

76
Q

A city council and park board announced joint plans to tear down some old buildings and erect a park. Before the contracts were made, in order to garner the greatest political benefit from such projects, the city council adopted an ordinance requiring that 35% of the work force of contractors working on city-funded projects be residents of the city.

One of the contractors working on the park project employed several people from the city, but he and most of his employees came from a town in a neighboring state that was a few miles west of the city. When the city projects inspector discovered that the contractor did not employ the required 35%, he told the contractor that if he did not hire a sufficient number of city workers within 20 days the contractor would forfeit the opportunity to work on the project. The contractor immediately filed an action in federal court seeking to have the employment requirement declared unconstitutional.

Of which party should the court should rule in favor?

A The city, because it is acting as a “market participant” here.

B The city, because there is a rational basis for favoring city residents here.

C The contractor, because the requirement interferes with his rights under the Privileges and Immunities Clause of Article IV.

D The contractor, because the requirement interferes with his Contract Clause rights.

A

C: The court should rule in favor of the contractor because the pursuit of a livelihood is a right protected by the Privileges and Immunities Clause, and the requirement here substantially interferes with that right. The Privileges and Immunities Clause of Article IV prohibits states and municipalities from discriminating against residents of other states. Not all discrimination is prohibited—only that which substantially interferes with important commercial activities or civil liberties. The Supreme Court has held that the right to pursue a livelihood is a right protected by the Privileges and Immunities Clause, and also has held that a requirement that private contractors on city projects employ a certain percentage of city residents substantially interferes with the right. [See United Building & Construction Trades Council v. Mayor of Camden (1984)]

(A) is incorrect because there is no market participant exception under the Privileges and Immunities Clause. The market participant exception arises from the Commerce Clause and is not appropriate in privileges and immunities analysis. [See United Building & Construction Trades Council v. Mayor of Camden, supra]

77
Q

Tell me about Congress’ power to investigate

A

The power to investigate to secure information as a basis for potential legislation is very broad, but the investigation must be for purposes within the scope of Congress’s power.

78
Q

Using a withdrawn guilty plea?

A

(a) Prohibited Uses. In a civil or criminal case, evidence of the following is not admissible against the defendant who made the plea or participated in the plea discussions:
(1) a guilty plea that was later withdrawn;
(2) a nolo contendere plea;
(3) a statement made during a proceeding on either of those pleas under Federal Rule of Criminal Procedure 11 or a comparable state procedure; or
(4) a statement made during plea discussions with an attorney for the prosecuting authority if the discussions did not result in a guilty plea or they resulted in a later-withdrawn guilty plea.
(b) Exceptions. The court may admit a statement described in Rule 410(a)(3) or (4):
(1) in any proceeding in which another statement made during the same plea or plea discussions has been introduced, if in fairness the statements ought to be considered together; or
(2) in a criminal proceeding for perjury or false statement, if the defendant made the statement under oath, on the record, and with counsel present.

79
Q

Accompanying the invoice on the boxes was a letter from the bulk retailer stating that the mouthpieces are compatible with two of the models of French horn, but that the retailer makes no warranties as to the compatibility of the mouthpieces with any other model of French horn.
Enough as an accommodation?

A

NO therefore if we are talking about UCC the sending of non conforming goods + this letter is not enough to make it a counter-offer.

80
Q

Can a dead person be defamed?

A

NO a dead person cannot be defamed

81
Q

Tell me the rules of non-conforming goods under the UCC?

A

UCC requires perfect tender, which means that the goods and their delivery must conform to the contract in every way or there is a breach. Here, the brewery breached the contract by delivering the beer late. When a delivery is nonconforming, the buyer may: (i) reject the goods and cancel the contract or sue for damages; or (ii) accept any commercial units, reject the rest, and sue for damages.

Once the goods are accepted, the buyer generally is bound on the contract and it is too late to cancel; however, the buyer retains the right to sue for damages for any nonconformity. When the tavern owner accepted the beer, he became bound under the contract to pay for the beer. Nevertheless, he can sue for damages that arose from the late delivery.

82
Q

A contractor agreed to build a plant for a manufacturer for $5 million, with $1 million paid in advance and the balance to be paid upon completion of the project. The contract required the contractor to use lighting fixtures from a specific company. Inadvertently, the contractor installed fixtures from a different company. The installed fixtures are generally considered to be of a slightly better quality than the fixtures specified in the contract. The mistake was not discovered until the manufacturer did a final inspection of the building. As built, the plant is worth $10,000 more than it would have been worth had the specified fixtures been used. It would cost the contractor $100,000 to replace the fixtures with the ones specified in the contract. Because of a downturn in the economy, the manufacturer no longer wants to move into the new plant and refuses to pay the contractor because of the breach regarding the light fixtures.

If the contractor sues the manufacturer for breach of contract, which of the following doctrines will be most important to a court’s decision?

A Perfect tender.

B Divisibility.

C Substantial performance.

D Quasi-contract.

A

C becasue D (restitution) applies only when a) there is no contract and b) when there has been an unjust enrichment by D.

Under the doctrine of substantial performance, contracts governed by the common law are enforceable despite minor breaches. The contract here—to build a manufacturing plant—is governed by the common law. In determining whether a breach is minor or substantial, courts look to whether the party received the substantial benefit of the bargain. Here, the manufacturer got a plant that was, perhaps, better than the one called for in the contract. Therefore, the contract will be enforceable under the doctrine of substantial performance. (A) is incorrect because the perfect tender doctrine applies only to contracts for the sale of goods—not to common law contracts.

D) is incorrect because quasi-contract provides a remedy in some situations in which a contract fails (or is absent) and the defendant would be unjustly enriched as a result. The doctrine is not applicable here because, as discussed above, the contract will not fail due to a minor breach.

83
Q

A man borrowed $5,000 from his colleague to purchase stock and agreed in writing to repay the loan on or before August 1.

On August 1, the man notified his colleague that he would be unable to pay back the $5,000. He told her that he could send her a check for $2,500 and that, in addition, he could give her an antique diamond ring that had been recently appraised at $2,200. The colleague liked the ring and agreed to accept it plus $2,500 in cash as payment for the loan.

On August 2, a courier delivered the ring and a certified check for $2,500 to the colleague. She took the check but told the courier to return the ring to the man. The man received the ring back the same afternoon. Meanwhile, the colleague deposited the check in her bank, and the next day filed suit against the man for $2,500. The man consulted an attorney as to whether he has a valid defense against his colleague’s suit.

Assuming there are no Statute of Frauds issues, what advice should the attorney give the man?

A The man has no defense against his colleague’s suit, because the amount of the debt was undisputed.

B The man has no defense against his colleague’s suit, because she properly exercised her right to enforce the original agreement by refusing tender of the ring.

C The man has no defense at law, but he may successfully defend in equity under a specific performance theory because the ring is unique.

D The man has the option of defending in equity under a specific performance theory or waiting until his colleague obtains a judgment against him and then suing her for breach.

A

The man has the option of defending in equity under a specific performance theory or waiting until his colleague obtains a judgment against him and then suing her for breach. He has either option available because his colleague is in breach of their accord agreement. An accord is an agreement in which one party to an existing contract agrees to accept, in lieu of the performance that she is supposed to receive from the other party, some other, different performance.

The accord must be supported by consideration, but the consideration is sufficient if it is of a different type than called for under the original contract, even if the substituted consideration is of less value. An accord suspends the right to enforce the original agreement. Performance of the accord (i.e., satisfaction) cuts off the parties’ rights to enforce the original contract and discharges the accord.

Here, the accord was supported by sufficient consideration because the man was giving a ring in lieu of some cash. The man’s duties under the accord were discharged when he timely tendered delivery of the ring and cash. By refusing the ring and filing suit for the part of the original debt that has not been paid, the colleague has breached the accord agreement. If a creditor breaches an accord agreement, the debtor has the option of either raising the accord agreement as an equitable defense in the creditor’s action and asking that it be dismissed, or waiting until he is damaged (i.e., until the creditor is successful in an action on the original contract) and then bringing an action at law for damages for breach of the accord contract.

(A) is incorrect because the amount of the debt does not have to be in dispute to have an enforceable accord, as long as there was some alteration in the debtor’s consideration, as discussed above.

(B) is incorrect because the colleague would have the right to enforce the original contract only if the man had breached the accord agreement. Here, the man’s tender of the ring discharged his duty under the accord agreement, precluding his colleague from suing on the original contract.

(C) is incorrect because the man has both a breach of contract remedy and an equitable defense option available to him. Also, whether the ring is unique does not affect his right to specific performance of the accord agreement; he is simply raising the agreement as an equitable defense to prevent the colleague from continuing with her suit on the original contract.

84
Q

In July of last summer a grape grower contracted with a winery to deliver “500 tons of premium quality pinot chardonnay grapes grown on my ranch.” The price was to be $1,000 per ton and delivery was to be on or before September 15. In August of the same year, the grape grower entered into an identical contract with a vineyard to sell 300 tons of premium quality pinot chardonnay grapes.

The grape grower completed his harvest by September 10 and had 800 tons of premium quality grapes. On September 11, an unexpected rain ruined 400 tons, and the grape grower notified the winery and the vineyard on that day that he would only be able to deliver 250 tons to the winery and 150 tons to the vineyard. On September 14, the vineyard purchased an additional 150 tons of premium quality pinot chardonnay grapes from a different grape farmer, one of several other available sources for premium quality pinot chardonnay grapes. These grapes along with the 150 tons from the grape grower gave the vineyard the 300 tons it needed.

On September 15, what is the winery’s legal position with regard to the grape grower’s failure to deliver the 500 tons of grapes required by his contract?

A If the winery has given the grape grower a written notice of termination, the winery will have the right to refuse to accept the 250 tons of grapes but will have no cause of action for damages against the grape grower.

B Even if the winery has given the grape grower a written notice of termination, the winery must accept the 250 tons of grapes and will have no cause of action for damages against the grape grower.

C Because the vineyard’s purchase establishes that it is possible for the grape grower to perform by obtaining additional grapes from other available sources, the winery may accept the 250 tons from the grape grower and recover damages for the grape grower’s failure to deliver the balance of the amount specified by the contract.

D Because the grape grower’s contract with the winery was entered into before his contract with the vineyard, the grape grower is bound to deliver the entirety of his grape crop to the winery.

A

A:
The winery may refuse the shipment if notice of termination is given but will not recover damages for breach.

This problem is governed by UCC sections 2-613, 2-615 and 2-616. When crops are destroyed in the case of a farmer who has contracted to sell crops from a designated tract of land, the loss may be governed either by UCC section 2-613 (casualty to identified goods) or section 2-615 (failure of presupposed conditions/commercial impracticability).

The result is the same under either section. Under section 2-613, if goods identified when the contract is made suffer casualty without fault of either party before the risk of loss passes to the buyer, the contract may be avoided.

If the loss is partial, the buyer may treat the contract as avoided or accept the goods with allowance from the price for the deficiency. The buyer does not have any further rights against the seller and thus cannot sue the seller for breach.

Under section 2-615, a crop failure resulting from an unexpected cause excuses a farmer’s obligation to deliver the full amount as long as he makes a fair and reasonable allocation among his buyers.

The grape grower has done this by allocating pro rata between the winery and the vineyard. Nevertheless, under UCC section 2-616, the buyer may either accept the proposed modification or terminate the contract. Under either provision, the winery is free to reject the 250 tons of grapes

85
Q

A patient went to a highly reputable dentist, who honestly told her that she needed a lot of dental work, some of it involving complex procedures. When she asked the dentist what the cost would be, he told her “about $3,500.” The patient agreed to use him as her dentist and he began her treatment. When the patient’s treatment was finished, the dentist sent her a bill for $4,100, explaining that the higher bill was because more expensive inlays were used, and that he carefully documented the cost of his materials and had sound medical reasons for his decision. The patient honestly believed that it was unfair for the dentist to charge her $4,100. Therefore, she sent the dentist’s invoice back to him, along with a check for $3,500. On the check the patient had clearly written in large letters “Payment In Full.” The dentist read the notation on the check and deposited it at his bank. The dentist made no notation of his own on the check other than his signature on the back as an indorsement. Two weeks later the dentist called his bank to make sure that the patient’s check had cleared. He then immediately filed suit against the patient for $600.

Is the dentist likely to succeed in his action against the patient for $600?

A Yes, because the dentist merely estimated the cost of the dental work to be $3,500.

B Yes, because the dentist can document that the precious metal inlays were medically necessary and that he charged a fair price for them.

C No, because there has been an accord and satisfaction of the original debt.

D No, because the patient’s duty has been discharged by an account stated.

A

C:
The dentist will not prevail because the contractual duty was discharged by an accord and satisfaction. An accord is an agreement in which one party to an existing contract agrees to accept, in lieu of the performance that she is supposed to receive from the other party, some other, different performance. Satisfaction is the performance of the accord agreement. The effect of this performance is to discharge both the accord agreement and the original contract. An accord and satisfaction may be accomplished by a good faith tender and acceptance of a check conspicuously marked “payment in full” where there is a bona fide dispute as to the amount owed. The dentist and the patient agreed that the dentist would perform the dental work needed by the patient in return for the patient’s promise to pay “about $3,500.” The facts indicate that, following the actual dental work, there ensued a good faith dispute as to whether the patient owed only $3,500 or the additional $600 as well. The patient in good faith tendered to the dentist a check marked “payment in full,” which notation the dentist saw before he deposited the check into his account. The dentist’s acceptance of the patient’s check gives rise to an accord and satisfaction with regard to the disputed original debt. Thus, the dentist is deemed to have accepted the amount tendered by the patient as full payment for the dental services performed, and the patient’s duty to pay for the services is discharged. (

86
Q

Can the court take judicial notice of birth dates?

A

Court will not take judicial notice of the birthdate. A birthdate (other than that of a famous person such as George Washignton) is not the type of fact that a court will not recognize as true without formal presentation of evidence (ex. presentation of a certified copy of a birth certificate).

Under the Federa; Rules courts may judicial notice of a) facts generally known within the territorial jurisdiction of the court or ii) capable of accurate and ready determination by resort to sources who accurate cannot reasonably be questioned.

Record of state or federal court judicially noticable.

Rained: ok
Indipendence Day: OK

87
Q

There drivers were involved in a three car accident in a city in the Middle District of State A. One of the drivers was severely injured and intends, in good faith, to file a negligence actions seeking $500,000 in damages against the other two in federal court. The plaintiff is a citizen of State A and resides in the Middle DIstrict. One defendant is a citizen of State B and resides in its Northern District. The other defendant is a citizen and resident of State C, a single district state. At the time fo the accident, the State C defendant was staying in a State A hotel for two months while econstuling on a construction project.

In what district or districts is venue proper?
A): the Middle District of State A only
B) The NOrthern DIstrcit of State B and the DIstrict of State C only
C) the Middle District of State A, the Northern District of State B, and the Distrcit of State C
D) None, the action may not be maintained in federal court because federal subject matter jurisdiction does not exist

A

A stupid

Federal venue in civil actions is proper in a) the district where any D resides, IF ALL D are residents of the state in which the district is located, and b) the district in which a substantial part of the events or omissions givigin rise to the claim occurred. Here, the defendants do not reside in the same state. Therefore venue cannot be based on the resident of the defendants. However, the accidentoccurred in the Middle Distrct of State A which is a proper venue under prong ii.

88
Q

A land owner gratuitously conveyed his interest in land to a friend by quitclaim deed. The friend promptly and properly recorded her deed. Si9x months later, the landowner conveyed his interest in the same land to an investor for his interest in these am eland to an investor for 50,000 by warranty deed which was promptly and properly recorded.

As between the friend and th investor, who has the superior right to title to the land?

a) the friend, regardless of the type of recording statute
b) the friend, because she recorded prior to the investor’s recording
c) the investor, regardless fo the type of recording statute
d) the investor, because it took by warranty deed rather than quitclaim deed

A

A
Because the firned recorded prior to the subsequent conveyance, she has the superior right to title regardless fo the typ of recording statute. A conveyance that is recorded can never be divested by a subsequent conveyance through operation of the recording statutes. By recording, the grantee gives a subsequent conveyance through operation of the recording statutes. By recording, the grantee gives constructive (or record) notice to everyone. Hence, proper recording prevents anyone from becoming a subsequent bona fide purchase. Because the landowner’s conveyance to the friend was recorded at the time of the landowner’s conveyance to the investor, the investor cannot previal. The investor will clearly lose under a pure race statute because the friend recorded first.

THe investor will also lose under notice an drace notice statute because the conveyance to the friend was recorded at the time of the conveyance to the investor.

The investor therefore had record notice and cannot claim the protection that these types of statutes provide for subsequent purchasers for value who take without.

89
Q

An online search engine corporation wishing to acquire a smaller website entity hired a law firm to perform an audit of its target for a bundled fee of 2.5 million. A first fee installment of 1 million was to be paid on completion of the data collection process, which was to be completed within three weeks. A second installment was to be paid on receipt of a valuation of the target website entity’s liabilities, due after another five weeks. GThe contract provided in part that “any amendments to this agreement must be in writing signed by both parties”/

On completion of data collection after 25 days, the law firm demanded payment of the first installment payment of 1 million. The client refused, but negotiations conducted between the parties resulted in an oral agreement that the parties resulted in an oral agreement that the client would pay 750,000 immediately and then the 1.5 million second installment as originally agreed, after valuation of all liabilities.

Was the oral agreement that the client pay 750,000 to the law firm after 25 days a valid modification of the original agreement?

a) yes because the SOD does not bar subsequent oral modification of a written agreement to which it is applicable
b) yes because contracts for services may be orally modified if consideration is present despite the existence of a no oral medication clasue
c) no because it was not in writing
d) no because it was not supported bu consideration

A

Remember that the oral agreement was a valid subsequent medication because the parties may orally waive the contract provision limiting amendment to a writing.

Therefore B correct, because contracts for services may be orally modified if consideration present despite the existence of no oral modification clasue.

90
Q

The father of a gidl whose horse crashed through a neighbor’s fence told the neighbor that he would have the fence repaired immediately at his cost. The father hired a local contractor to do the work. The contractor did a poor job and did not fix the fence properly.

Assuming that there was an enforceable agreement between the father and the neighbor, would the neighbor have an action against the contractor?

a) no because there was no agreement between the contractor and the neighbor with regard to the repairs on the fence
b) no because the neighbor was only an incidental beneficiary of the contract between the contractor and the father
c) yes because the neighbor was a creditor beneficiary of the contract between the contractor and the father
d) yes ebacsue the neighbor was a donee beneficiary of the contract between the contractor and the father.

A

C) the neighbor as third party creditor beneficiary of the agreement to repair the fence has an action against the contractor.

A third party is a creditor beneficiary of a contract if the promisee’s primary intent in contracting is to discharge an obligation to the third party. Here ht efahter bargained with the contractor to do repair work on the neighbor
s fence in discharge of an obligation that the father had to the neighbor. THerefore the neighbor was the thrid party creditor beneficiary of the agreement to repair the fence and could sue the contractor on this agreement.

91
Q

A D was indicted by a grand jury on counts of murder involuntary manslaughter and aggravated robbery. At his arraignment the trial court, over the prosecution’s objection accepted the defendant’s guilty pleas to involuntary manslaughter an d aggravated robbery. The court entered a conviction on those offenses and then granted the defendant’s motion to dismiss the murder charge on double jeopardy grounds. The prosecution appealed the court’s dismissal of the murder charge, arguing that it should be permitted to try the defendant on that charge.

How should the appellate court rule?

a) reverse because double jeopardy does not bar trial on the murder charge
b) reverse because there judge improperly accepted a plea bargain over the prosecution’s objectiosn
c) affirm, because the defendant has been convicted of a lesser included offense
d) affirm, because collateral estoppel precludes trial and conviction on the murder charge after it has been dismissed.

A

A:

The appe;ate court should reverse because prosecution of the murder charge does not violate the Double Jeopardy Clause. The Fifth Amendment right to be gree from double Jeopardy for the same offense precludes retrial for a greater offense once jeopardy has attached for a lesser included offense, as well as the imposition of multiple punishments for the same offense.

However, a state may continue to prosecute a charge offence, despite the defendant’s guilty plea to a lesser included or “allied” offense arising from the SAME incident.

The acceptance of a guilyu plea on the lseer included offenses while eht charges not eh greater offence remain pending has none of the implications of an “implied acquirral” that results from a guilty verdict on lesser included offenses rendered by a jury charged to consider both greater and lesser included offenses. Nothwithstanding the trail court’s acceptance of the defendant’s guilyu pleas, the defendant is not entitled to use the Double Jeopardy Clause as a sword to prevent the prosecution from continuing to pursue the remaining charge of murder. Hence, the trial court should be reversed.

92
Q

A licensed real estate broker and a homeowner entered into a written listing agreement in which the homeowner entered into a written listing agreement in which the homeowner promised, among other things, to pay the real estate broke obtained a buyer ready, willing and able to purchase it. The homeowner’s home was listed for 180,000 in a service made available to real estate professionals.

A prospective buyer, after going to the real estate broker’s office nad viewing the homeowner’s home, submitted a written offer to purchase the home for $180,000. The homeowner rejected this offer by not accepting it within the state period. The buyer brings an action against the homeowner for specific performance, seeking to compel himto sell the home.

What is the probable outcome of this litigation?

a) the homeowner will win, because no writing or writings constitute a memorandum sufficient to satisfy SOF
b) the homeowner will win, ebacsue the buyer;s remedy at law is adequate
c) the buyer will win because he is a third party beneficiary of the agreement between the homeowner and the real estate broker
d) the buyer will win because there is a memorandum that satisfied the Statute of Frauds.

A

A:

The buyer cannot obtain specific performance against the moeowner because the absence of a written memorandum signed by the homeowner and containing the essential terms of an agreement between the buyer and the homeowner means that there is no enofceable contract.

To obtain specific performance there must be an enforceable contract between the parties.

93
Q

A testator wanted to give his home to his brother. The testator executed a warranty deed conveying the home to his brother. The testator then wrote a letter to his brother saying, “Dear brother, my home is now yours”. He put the letter in an envelope and wrote the following on the outside of the envelope and signed it: “Cousin you are to give this deed to my brother when I die. Until then, you should safeguard this envelope and the documents inside”. The testator delivered these items to his cousin and continued to live in his home by himself two years later the testator executed a will leaving all of its property to his sister. Sever years later, the testator died. Shortly thereafter, the causing delivered the envelope containing the deed to the testator’ brother who promptly recorded the deed. The testator’s will has been admitted to probate and his executor has brought an appropriate action against the brother to determine the title to the testator’s home.

For which party should the court rule?

a) the system because the deed was not effectively delivered before the testator died
b) for the sister because the deed was not recorded before the grantor died
c) for the brother because the deed effectively conveyed title when it was executed
d) for the brother, ebacsue the testator no longer owner his home hwne he died.

A

YES. Because testator did not retain a right to revoke the escrow his delivery of the deed to his cousin completes the conveyance. Title passes to the brother automaticall on the testato’r death and “realtes back” to the date of delivery to the cousin. Thus, the testator’s home was not his property at his death and it cannot pass to the sister.

94
Q

A federal statute designed to stop organized crime enumerated certain activities as crimes and provided that, in addition to charging these activiters as the crimes they constitute, the activities would also constitute the criminal act of intentional furtherance of the goals of organized crime. AMong the enumerated activities was the interstate distribution of cocatin. The statute’s constitutionality has been upheld by the Supreme Court.

The D was arrested by federal agents after having driven a truck containing cocaine from Florida to Illinois, where he delivered his illicit cargo as directed. At trial, the defendant is convicted of interstate distribution of cocaine in violation of federal law, and convicted of a violation of federal statute above.

How may the defendant be sentenced?

a) under either statute but not both
b) under both statutes
c) only under the statute that carries a lesser maximum sentence
d) only under the statute that carries a greater maximum sentence.

A

B:
the defendant may be sentenced under both statutes. Double jeopardy does not prohibit the imposition of cumulative sentences for two or more statutorily defined offenses specifically intended by the legislature to carry separate punishments, even though constitution the “same” crime under the Blockburger test (each offense does not require proof of some additional fact that the other does not) when the punishments are imposed at a single trial.

Absent a clear intention, it is presumed that multiple punishments are not intended for offenses constituting the same crime under Blockburger.

HERE IT IS CLEAR THAT CONGRESS IN ENACTING THE STATUTE, INTENDED THAT CERTAIN OFFENSES USCH AS INTERSTATE DISTRIBUTION OF COCAINE BE SUBJECT TO SEPARATE PUNISHEMTNS.

95
Q

Where a survey of land and a plat conflict which one wins?

A

The survey.The plat is onyl intended to be a representation of the actual survey as made on the land itself. THe play is in the nature of a certified copy of an instrument that will be controlled by the original. Where a survey as made and marked on the ground conflicts with the play the survey prevails.

96
Q

A young child was using real fireworks to blow up his toy tanks in his backyard during a pretend game of war while his father was in the house. The child was severely injured when one firework exploded in his hand. The childn’s mother, as guardian ad litem, brought an action against the fireworks manufacturer to recover for the child’s injuries and medical expenses. THe fireworks manufacturer defended on the ground that the child’s father was nelgigent in supervising the child. THe jurisdiction has abolished parent child tort immunity.

What effect will any negligence on the father’s part have?

a) no effect, because the father’s negligence is not imputed to the child
b) no effect because the jurisdiction has abolished parent child tort immunity
c) reduce the mother’s recovery because the mother’s action is derivative of the child’s cause of action
d) recude the mother’s recovery, because the jurisdiction has abolished parent child tort immunity.

A

A
The father’s negligence will have no effect because a parent’s contributory negligence is not imputed to the child. Nor is the father’s negligence imputed to the mother because the contributory negligence of one spouse is not imputed to the other spouse.

B and D are incorrect beuacse the parent child immunity would be relevant only if the child were suing a parent for negligence - it is inapplicable here

97
Q

Another larceny question ( : ) )
A defendant got into a fight with his former roommate over some money that the defendant claimed his old roommate owed fo rsome long distance telephone calls. When his old roommate refused to pay the money, the defendant took two tickets that the roommate had purchased for a play off college basketball game, intending to five them back the day after the fame. The defendant is charged with larceny.

Will the defendant be found guilty?

a) no because he intended to return the tickets to his old roommate
b) no because he believed that his old roommate owed him money’c) yes because he intended to deprive his old roommate of the value of the tickets
d) yes because the intent to return is never a good defense.

A

C: the defendant will be found guilty of larceny. The only value the tickets will have to anyone is if they are used d for admission to the basketball game. Consequently beaucse the defendant did NOT itnend to return the tickets to hos old roommate until after the fame, the defendant intended to depirve his old roommate of the value of the tickets for an unreasonable amount of time, and thus most likely would be found fuitly of larceny.

B is a tempting answer but it is to the most likely. There are no facts that indicate the relative value of the tickets to the money that the defendant claims his old roommate owes him.

98
Q

Contract - payment of debts - how does it work?

A

Payment of a smaller sum that due will not be sufficient consideration for a promise by a creditor to discharge a debt unless the consideration is in some way new or different (ex. payment before maturity or to one other than the creditor) or the amount of debt is subject to an honest dispute (and so the parties are giving up the right to litigate the amount which is consideration sufficient to support a modification in and of itself.

99
Q

MIsrerepresentation elemetns?

and what if I say that I assembled this car with the best assembly ever? misrep?

A

To establish a prima facie case of intentional misrepresentation (deceit), a plaintiff must establish a) a misrepresentation made by the defendant, b) scienter, c) an intent to induce plaintiff’s reliance on the misrepresentation, d) causation (actual reliance), e) justifiable reliance and f) damages.

And no, not “reasonable reliance

100
Q

(*)TWo police officers noticed a car was weaving and generally being driven in an erratic manner. They pursued the vehicle and curbed it. When the driver emerged from the car, he was obviously intoxicated. The officers arrested him and put the driver in their squad car to take him to the station house. The driver’s wife was in the passenger seat and she followed the police in th ecar to the local precinct.

As the driver was being booked on officer took a standard police investor sheet and began searching the car. Beneath the passenger seat he found the passenger’s purse. He opened the purse and found a plastic zip-lock bag containing a small amount of marijuana. The passenger was charged with possession of marijuana. Prior to trial her attorney moved to suppress the admission of the marijuana seized from the passenger’s purse into evidence.

Should the court rule favourable on the motion?

a) yes because when conducting a search incident to an arrest the police may not open a closed container
b) yes because the police lacked probable cause to search the passanger’s purse
c) no because the search was incident to the lawful arrest of the driver
d) no because the marijuana was discovered during the course of a valid investor search.

A

B:
The motion to suppress the evidence will be granted because the police did not have probable cause to search the car. When the police place the driver of an automobile under arrest, there are a number of alternatives with respect to a search of the car: a) they may conduct a search of the passenger compartnermet incident to arrest if the search is contemporaneous with the arrest and either the arrestee may still access the vehicle or the police reasonable believe that the vehicle contains evidence of the crime for which the arrest was made. Here the search was not contemporary and nothing indicates that either of the other two conditions was satisfied.

ii) if the police have probable cause to search the car. (reasonable grounds for believing that a legitimate item of seizure is in the car) a search of the entire car can be made without a warrant. THe search based on probable cause can be made at the time of the arrest or at a later time.

In this question the search of the car at the police station would have been valid IF the police had probable cause to search the car. The question does not provide any facts that could form the basis for probable cause to search. Thus the motion to suppress will be granted because of the lack of probable casue.

iii) If the police take the car under their control for an administrative reason (such as car to get off the highway) they can “investor” the items in the car under certain circumstances. In this question the car had not been impounded by the police nor would it need to be because the passenger could drive it home.

101
Q

Tell me the treatise exception.

A

Under the Federal Rule, information in treatises can be read into evidence if the treatise is relied upon by the expert

OR is called to his attention during cross examination and established as reliable by the witness another expert or judicial notcie

102
Q

rule on negligence on person who designed something that was dedicated.

A

The prevailing rule is that in performing services of designing and constructing improvements on property eventually deeded to the city a developer must accept responsibility fro prededication negligence.

103
Q

When someone does NOT reply a discovery request at all, MUST the court impose sanctions?

A

THe court mut impose sanctions on the beneficiary or her lawyer. If a party fails to respond to a request for inspection, the opposing party may move for immediate sanctions. The motion must certify that the moving party has made a good faith attempt to obtain a response.

The court has discretion to choose from a variety of sanctions, but the court must require the party that failed to respond to discovery, the party attorney or both to pay the movant’s reasonable expenses caused by that failure.

104
Q

Partial condemnation case.

A landlord leased a 40 acre tract o fland to a tenant for 15 years. After 5 years had a=expired the government condemned 15 acres of the poreprty fo rroad construction and allocated the compensation award to the landlord and the tenant according to their respective interest. It so happened however that the tenant had used the 15 acres taken by the government to store vehicles necessary in the tenant’s work. The tenant knew of no other place nearby where he could store the vehicles. There is no applicable statute in the jurisdiction where the property is located nor any provision in the lease relatin to the condemnation. The tenant quir possession claiming that he could no l0nger live on the premises if he could not park the vehicles needed in this work close to where he lived. The landlord brought suit against the tenant to recover rent.

Is the landlord likely to prevail?

a) yes because the releatopisnhip of landlord and tenant was unaffected by the condemnation thus leaving ght tenant till obgliated to pay rent
c) no ebacusee there has been a breach of the implied covenant of quiet enjoyment by the landlord’s inability to provide the tenant with possession of the whole of the property for the entire temr.

A

A
The landlord liekly will prevail. In a partial condemnation case the landlord tenant relationship continues a does the tenant’ s obligation to pay the entire rent for the remaining lease term

C is wrong because the covenant of quiet enjoyment can be breached only by actions of the landlord and not those of a third party such as government.

105
Q

A state suffering from a sever loss of tax revenues due to an initiative that cut state sales in half, enacted legislation that ended cost of living increases in all state employees’ pensions. A state organization of employees brought suit against the appropriate state official in the federal court to reinstate the increase.

What is the most likely result?

a) the em[plpoyees’ organization will prevail because the statute violates the prohibition against the impairment of the obligations of contracts by a state
b) the employees’ organization will prevail if it can show that the statute violates the state’s constitution

A

A: the employees organization likely will prevail. The Constitution prohibits the impairment of contractual obligations by a state except in certain narrow circusmtnaces. THe sort of “emergency” normally required for such state action is arguable present here, given the loss in tax revenues. But it is unliekylt that the state would prevail because the termination of annual cost of living adjustments is permanent and appears to be the sort of self interest driven choice to reduce the state’s contractual burdens that the Court has found suspect in comparable case.s

106
Q

A state suffering from a sever loss of tax revenues due to an initiative that cut state sales in half, enacted legislation that ended cost of living increases in all state employees’ pensions. A state organization of employees brought suit against the appropriate state official in the federal court to reinstate the increase.

What is the most likely result?

a) the em[plpoyees’ organization will prevail because the statute violates the prohibition against the impairment of the obligations of contracts by a state
b) the employees’ organization will prevail if it can show that the statute violates the state’s constitution

A

A: the employees organization likely will prevail. The Constitution prohibits the impairment of contractual obligations by a state except in certain narrow circusmtnaces. THe sort of “emergency” normally required for such state action is arguable present here, given the loss in tax revenues. But it is unliekylt that the state would prevail because the termination of annual cost of living adjustments is permanent and appears to be the sort of self interest driven choice to reduce the state’s contractual burdens that the Court has found suspect in comparable case.s

B is wrong because federal coruts do not have jurisdiction to decide questions regarding an individual state’s own constitution.

107
Q

Party A asks party B to summarize what happened. A later asks for what it said to B. Ok or privileged?

A

A party may obtain without a court order and without showing need and hardship a copy of any statement or recording previously made by that party.

108
Q

A state passed a law requiring that anyone holding himself out to be a private investigator in the state must be licensed by the state. Licensure requirements included a thorough background check into the person’s criminal record and mental health. It also required passing a test on ethical obligations of a private investigator. Finally, the investigator was required to sign a two-part oath. Part one was a loyalty oath, which stated: “I solemnly swear (or affirm) that I will be loyal to the United States and to the state and will uphold their Constitutions.” Part two stated: “I solemnly swear (or affirm) that I am not now a member of any organization that advocates illegal acts, nor will I become a member of any such organization while I am a licensed private investigator in this state.”

An experienced investigator with a master’s degree in criminal justice administration applied for a private investigator’s license. He easily passed both background checks, but he refused to take the oaths, claiming that they inhibited his freedoms of speech and association as guaranteed by the federal Constitution. The state professional licensure board denied him a private investigator’s license solely on the basis of his refusal to take the oaths. The investigator sued in federal court to require the state to grant him a license and to strike down the oath requirements in the licensure statute.

What is the court’s likely ruling?

A Both the loyalty oath and the membership oath are constitutional.

B The loyalty oath is constitutional, but the membership oath is unconstitutional.

C The membership oath is constitutional, but the loyalty oath is unconstitutional.

D Neither the loyalty oath nor the membership oath is constitutional.

A

The loyalty oath is constitutional, but the membership oath is unconstitutional. The First Amendment protects the rights of association and speech. State infringements on these rights must serve a compelling government interest, unrelated to the suppression of ideas. In the area of public employment, neither standards of conduct nor loyalty oaths may be vague or overbroad. Precision is required because of the potential chilling effect on First Amendment rights. Here, the investigator is not applying for public employment, but the state is requiring him to take the oaths to obtain a state license, which involves the same First Amendment issues.

The loyalty oath is virtually identical to oaths that have been held to be constitutional. The state has a compelling interest in seeing that both constitutions are upheld and there is nothing vague about the oath. Oaths similar to the membership oath, however, have been struck down as overbroad.

The state has a compelling interest only in preventing knowing membership with the specific intent to further unlawful aims.
Persons cannot be denied a license because of mere membership in a particular group, and the state statute here addresses mere membership. Thus, (B) is correct

109
Q

A group of students held a demonstration against the proliferation of weapons in space in one of the city’s parks. The defendant spoke at the demonstration, and to make a point during her speech, she walked over to one of the trash barrels and dumped the contents out on the ground. As she did so, she told her listeners, “This is what outer space is starting to look like, cluttered with the trash of nuclear weapons.” The meeting broke up 15 minutes later. The students left the park peacefully, but no one bothered to pick up the trash or right the overturned barrel. The defendant was arrested pursuant to the city’s littering ordinance, which carried fines of up to $500 and was strictly enforced. She was convicted and fined $500. The defendant brings an appropriate appeal to have her conviction set aside on constitutional grounds.

Is the defendant likely to succeed?

A No, because the anti-littering ordinance furthers an important governmental interest and is not aimed at communication.

B No, because the defendant was convicted for her conduct rather than for what she said.

C Yes, because the defendant’s conduct constituted symbolic speech.

D Yes, unless the city can prove a compelling interest in its anti-litter laws.

A

A

The anti-littering ordinance will be upheld because it furthers an important government interest unrelated to the content of the communication and is narrowly tailored to the furtherance of that interest. As a general rule, conduct that is intended to communicate is not immune from reasonable government regulation, even though it takes place in a public forum such as a park. The noncommunicative impact of speech-related conduct in a public forum can be regulated to further an important government interest independent of the speech aspects of the conduct as long as the incidental restriction on the ability to communicate that message is narrowly tailored to further the interest in question, so that alternative channels for communicating the message are available. The prevention of litter, as a means of maintaining public facilities in usable condition and protecting property values, is an important enough government interest to allow some type of regulation. The ban on littering is narrowly tailored to accomplish its purpose, unlike, for example, a ban on distributing leaflets that may end up on the ground. The regulation probably would not have precluded the defendant even from dumping the barrel if she had picked up the trash after her speech was over.

110
Q

A woman belonged to an extremist political group that advocated the overthrow of the United States government. Among the woman’s grievances against current government was a belief that young people were being brainwashed by public school curricula. To end this practice, the woman decided to run for the local school board in the next general election so that she would have a voice in future curricula decisions. The election laws of the city where the woman resided required all candidates for the school board to sign a loyalty oath and file it with the town clerk in order to have their names placed on the ballot for the next school board election. When the woman arrived at the clerk’s office to file her candidacy, she was informed of the loyalty oath requirement and was shown the oath. The woman read the oath and objected to the requirement that she sign it.

Which of the following statements is correct as to whether the city may constitutionally require the woman to sign the oath in order to have her name placed on the ballot?

A The loyalty oath requirement is valid if the candidate must merely affirm that she does not advocate the violent overthrow of the state or federal governments.

B The loyalty oath requirement is valid if the candidate must merely affirm that she will oppose the violent overthrow of the state or federal governments.

C The loyalty requirement is valid if the candidate must merely affirm that she has never been a member of any organization dedicated to the violent overthrow of the state or federal governments.

D No oath is constitutionally permissible as a condition for ballot qualification.

A

The loyalty oath is valid if it merely requires the candidate to affirm that she will oppose the violent overthrow of the government. The government may require employees and other public officers to take a loyalty oath. Such an oath must not be overbroad, so as to prohibit constitutionally protected activities, nor can it be vague. An oath has been upheld that requires state employees to oppose the overthrow of the government by force, violence, or by an illegal or unconstitutional method. [Cole v. Richardson (1972)] This type of oath (which is virtually the same as that set forth in (B)) was deemed to be similar to an oath requiring the taker simply to commit herself to live by the constitutional processes of our system. (D) is incorrect because, as noted above, a requirement for a loyalty oath as a condition of holding public office or employment is valid, as long as the oath is within constitutionally acceptable limits. (A) is incorrect because it has been held that a loyalty oath that disavows advocating the violent overthrow of the government as an abstract doctrine is invalid. The First Amendment prohibits statutes regulating advocacy that are not limited to advocacy of action. [Communist Party v. Whitcomb (1974)] The oath in (A) appears to be overbroad in that it regulates mere advocacy of violent overthrow of the government, rather than regulating only advocacy of action. Therefore, the oath is invalid. (C) is incorrect because only knowing membership in an organization with specific intent to further unlawful aims is unprotected by the First Amendment. [Keyishian v. Board of Regents (1967)] The oath in (C) would in effect punish mere membership in an organization, without showing knowledge of, or a specific intent to further, any unlawful aims of the organization.

111
Q

A plaintiff is suing members of the police department in federal court after receiving a near fatal beating in jail, alleging a violation of his federal civil rights.

Which of the following items of evidence will the court be likely to admit despite timely objections by the opposing attorney?
A) testimony by a witness for the plaintiff. who was locked up in an adjoining cell, that the plaintiff was in fact beaten by the defendant police officers, objected to on grounds that calling this witness constitutes an unfair surprise

B) testimony by an expert witness who will affirm the testimony of a previous expert that the injuries suffered by the plaintiff were inconsistent with injuries likely from the alleged police beating, objected to on grounds that it will unnecessarily present cumulative evidnece

C) introduction of the bloodstained shirt that the plaintiff wore on the might of the beating, objected to on grounds that it will create a danger of unfair prejudice

D) testimony by a police officer that, because the plaintiff had drug-related track marks on his arms and exhibited symptoms of hepatitis, the officers feared that he might have hapetitis, and consequently would not have beaten him for hear of being infected, objected to on grounds that it may conduse the issues or mislead the jury.

A

B:
The search of the glove compartment would not be valid if the officer had placed the driver under arrest and secured him in the back of the squad car. After arresting the occupant of a vehicle, the police may search the interior of the vehicle incident to the arrest if a) the arrestee is unsecured and still may ain access to the interior of the vehicle or b) the p;police reasonably believe that evidence of the offense for which the person arrested may be found in the vehicle.

Here, if the officer secured the arrested driver in th ebacl of the squad car the first basis for permitting the search incident to arrest no longer app;ies (and there are no facts to support the second basis). Hence, there would be no ground under (B)to support the search of the glove compartment.

A) is incorrect because under the stop and frisk exception an officer may order the occupant of a vehic;e out of the car and search the passenger compartment of the officer reasonably believes that the occupant may be dangerous

112
Q

A drug dealer was persuaded by his public defender to plead guilty to possession of marijuana. even though the drug dealer believes that the seizure of the marijuana was illegal. He pleaded guilty while protecting that he was innocent. The judge accepted the plea after determining that it was voluntary and intelligent.

After the drug dealer was released on bail prior to sentencing he was advised by a civil attorney that he could sue the arresting police officer to recover damages for an illegal search and seizure of the marijuana.

Is the civil attorney correct?

a) no because the guilty plea was voluntary
b) no because a guilty plea is binding whether or not it was voluntary
c) yes because the drug dealer protested his innocence
d) yes beuacse the drug dealer did not waive his 4th A claims when he pled guilty

A

D)

THe civil attorney is correct. The Supreme Court has rules that a defendant’s guilty plea neither admits the legality of the incriminating search nor waived Fourth Amendment claims in a subsequent civil damages action challenging the constitutionality of the incriminating search.

Therefore, although the drug dealer pled guilty, he may still bring civil claims challenging the constitutionality of the search performed by police officers.

A) is wrong. If the guilty plea was not voluntary, the plea itself would be invalid by it would have no bearing on the drug dealer’s ability to bring a civil claim for 4th A violations

113
Q

In a state where gambling is legal a professional gambler ran up a tab of $50,000 at his favourite casino. Pursuant to a longstanding agreement between the gamb ler and the casino, once the gambler’s tab reached $50,000 he was required to repay the debt in give monthly installments of $10,000 before putting any additional charges on his tab. After making three repayments, the gambler approached the casino owner and offered and immediate payoff of $15,000 in cash as payment in full. The casino owner had a cash flow problem and needed the money, so he agreed. The fambler made the cash payemtns of $!5,000 that same day. A few day later, the casino owner demanded $5000 from the gambler.

Dowes the casino owner have a right to collect $5000 from the ganb ler?
a) yes because the gambler hadmpreexisting duty to pay the full $50,000
b) yes because the casino owner acted under duress when he accepted the immediate payoff of $!5,000 in cash as payment in full for the gambler’s debt
c) no, because there was discharge by release
D) no because there was an accord and satisfaction.

A

D) the casino owner cannot collect from the gambler. There was a valid accord and satisfaction, discharging the gambler’s obligations under the contract. An accord is the agreement under which one party agrees to accept some other, different performance from the other party that he would have received under the existing contract. An accord, like other contracts, must be support by consideration. The casino owner’s willingness to take ea lesser amount in exchange for an earlier repayment of a mutually agreed upon amount is sufficient as mutual consideration. Payment of a smaller sum that due is sufficient consideration for a promise by a creditor to discharge a debt because the consideration was DIFFERENT (ex. immediate payment before the last payment was due). Thus there was consideration supporting the accord. The satisfaction of the accord occurred on the payment of the $15,000 cash. Satisfaction discharges both the accord and the original contract.

114
Q

(PLEASE NOTE!)
A buyer filed a breach of contract action against a seller in federal district court. The buyer alleges that the two parties had a contract under which the seller agreed to sell to the buyer a specified number of widgets at a specified price. The seller alleges that the parties had negotiated a possible sal ebut never reached an agreement. The seller thus denies the existence of a contract. The buyer spent several hundred dollars retrieving from its computer archives the email communication that took place between it and the seller the buyer intends to use the emails as evidence that the parties did in fact have a contract. The retrieved emails currently are saved on a computer hard drive at the buyer’s office.

To what extent may the seller obtain discovery regarding the emails?
A) the buyer must provide the seller a copy of the email as part of the buyer’s initial disclo0sures
B) although the buyer is not required to produce electronically stored information in its required disclosures, the seller may obtain a copy of the email by serving an appropriate request
C) the buyer may refuse to produce the emilas if it determines that they are not reasonably accessible
D) because the buyer gained access to the emials in anticipation of litigation or for trial the emails are subject to a qualified immunity from disvoery pursuant to the work product doctrine.

A

A) hte buyer must provide a copy of the emials as part of its initial disclosures. Without waiting for a discovery request, a party must provide to the other parties copies or descriptions of electronically stored information that is in the disclosing party’s oissesuib it cibtrik and that get discloaing party may use to support its claims or defenses

115
Q

Can a member of Congress (ex. Senator) challenge a law authorizing Presidnet to exercise a line item veto?

A

NOPE. The Senator will not success beacsue she lacks standing to challenge the statute. The Supreme Court has held that member of Congress lack standing to challenge a law authorizing the President to exercise a line item very (such as the statute her), reasoning that the injury is not concrete and personal but rather is institutional in that it is shared by all members of Congress.

116
Q

A member of an out of town yacht club agreed to buy a luxury boat from a local yachtsman, a boat which the parties referred to as “the Lady boat” throughout their preliminary negotiations. They yachtsman owner two boards – “Lady Be Good” and “Lady Luck”. The yachtsman’s intent was to sell “Lady Luck”, while the club member intended to buy “Lady Be Good”, which the club member knew was once owned by the club member’s favourite celebrity. THe club member was unaware of the existence of “Lady Luck”. The written contract for the yachtsman/s “Lady” board included a sale price of $100,000. When the yachtsman delivered “Lady Luck” to the club member’s slip, the club member saw that it was not the boat he had intended to purchase and refused to accept delivery.

If the bluc member sues the yachtsman for specific performance to compel him to deliver “Lady Be Good” and the yachtsman countersues to compel the bluc member to accept delivery of “Lady Luck” who should prevail?
A) the yachtsman beacuse in the case of a latent ambiguity in the contract, the intent of the seller controls
B) the club member, beacuse the yachtsman knew that he had two “Lady” boats at the time the contract was formed
C) Neither party, because an ambiguity existed at the time the contract was formed, and the parties fid not intend the same meaning
D) neither party, beacsue meony damages would be sufficeint

A

B ‘cause the other party knew

117
Q

You go to friend’s house who shows you a stolen pistol. When he is not looking you put it into your pocket. Larceny?

A

YUP!

Larceny is a crime against possession NOT ownership.

118
Q

Can Congress include in a statute that someone had standing?

A

NO Congress can’t just create standing, we still have to go through everything.

119
Q

A resident of State B was injured in the Southern District of State A when her car was struck by a large transport truck. At the time of the accident the truck driver was acting in the course of his employment for a ;are retail corporation, which owner the truck. The plaintiff intends to file a negligence action infederal district court against both the truck driver and the retail corporation. The truck driver is domiciled in the Central DIstrict of State A. The retail corporation is a State C corporation but has its principal place of business ina city in the District of State D. The corporation operates close to 100 stores in a number of states, including 20 stores in the Southern District of State A.

In which federal district court(s) is venue proper?
A) the Southern District of State A only
B) the Central District of State A and the Southern District of State A only
C) the Central District of State A, the Soutehrn District of State A and the District of STate. D only
D) the Central District of State A, the Southern District of State A< the District of State C and the DIstrict of State D

A

B:
Venue is proper in the Central District of State A and the Southern District of State A. Federal venue in civil actions is proper in a) the district where any defendant resides, if all defendants are residents of the state in which the district is located, and b) the district in which a substantial part of the events or omissions giving rise to the claim occurred. If there is NOT district anywhere in the US that satisfied a or B, the action my be brought in a judicial district in which any defendant is subject to the court’s personal jurisdiction with respect to such action. A corporate defendant is deemed to reside in each district with which it has sufficient contacts to justify personal jurisdiction with respect to the action. The retail corporation thus resides, among other places, in the Southern Distrct of State A, where it is subject to personal jurisdiction with respect to the action based on a specific jurisdiction theory. (Its commission would be used in the “fairness and reasonableness” assessment).

The truck drive is domiciled in the Cnetral District of State A. Given that both defendants reside in State A, any State A district an incorrect answer choice in that the truck driver’s domicile in the Central District is not taken into account.

120
Q

Requiring a prayer before a legislative meeting by calling religious leaders from all religions ok?

A

While it may seem that starting a legislative meeting with a prayer would not pass the Lemon test, the Supreme Court has held that because of the long history of legislative prayer in America, such prayers do not constitute an establishment of religion, and it is permissible for a legislative body, including a municipality, to invite members of the clergy to begin session with a prayer.

121
Q

(*)
The plaintiff and defendant were in a two car accident. The plaintiff filed a negligence action against the defendant in federal district court, seeking compensatory damages. After the accident but before the action was filed, the defendant’s attorney spent $25,000 in investigation fees to determine the identity of and track down a particular eyewitness to the accident. He did so exclusively to get ready for the anticipated litigation.

Must the defendant reveal the identity of the eyewitness to the plaintiff in discovery?
A) YES, as part of his initial disclosures regardless of whether the witness’ information is favourable to the plaintiff or defendant
B) Yes if either the defendant might call the eyewitness as a witness at trial or the plaintiff requests the identity of the eyewitness in a proper discovery request
C) no as longas it is possible fo the plaintiff and his attorney to discover the identity of the eyewitness through their own investigation
D) no because the information was developed by the defendant’s attorney and is protected by the attorney client privilege

A

B)
The defendant must reveal the identity of the eyewitness if either the defendant may call the witness at trial or if the plaintiff requests the eyewitness’s identity. As part of required initial disclosures, a party must provide the identities of individuals likely to have discoverable information that the disclosing party may use to support its claim or defenses. Therefore , if the defendant may use the eyewitness to support his claims or defenses, he must disclose the eyewitness’s identity to the plaintiff. If the defendant is not using the eyewitness to support his claims or defenses he must nevertheless disclose the eyewitness to support his claims or defenses he must nevertheless disclose the eyewitness’s identity if the plaintiff properly requests it because discovery may be had of any nonprivileged matter that is relevant to any party’s claim or defense including individual switch knowledge of any discoverable matter.

122
Q

RAP !!

A landlord leased her hom to a tenant for 10 years. The lease provided “on payment of the purchase price of 100,000 during the lease term or any renewal term, the landlord covenants and agrees to convey the said home to the tenant, his successors and assigns, by a recordable instrument having good and marketable title in fee simple”. Five years later, the tenant assigned the remainder, of the lease term to a friend with the landlord’s consent. Six months ago, the friend sought to exercise the option to purchase the home for 100,000 but the landlord refused. The friend brought an appropriate action for specific performance. The jurisdiction in which the home is located follows the common law RAP.

For whom should the court rule?
A) the landlord because the option vioaltes the RAP
B0 the landlord because the option is not assignable
C) the friend beacse the option is enforceable
D) the friend because he exercised the option within the perpetuities period.

A

C:
The court shoudl rule for hte friend because the option is enforceable. The RAP provides that no interest in property is valid unless it mus vest, if at all, no later than 21 years after one or more lives in being at the creation of the interest. OPtions to purchase generally are considered to be assignable and thus subject to the Rule, so that an option that might be exercised later than the end of the perpetuities period is void. however, the Rule doe snot apply to options connected to leasehold. Thus the option does not violate the Rule and A is incorrect.

123
Q

(LOOK!)
On March 1, a widget manufacturer and a retailer entered into a written contract whereby the manufacturer agreed to sell and the retailer agreed to buy 10,000 widgets at a price of $10,000. Due to slow sales, the manufacturer was operating its factory at only 50% capacity and had ample inventory on had. Delivery and payment was set for May 1. On April 1, the retailer told the manufacturer that he had no need or the widgets agter all and would not accept delivery of them on May 1. After notice to the retailer, the manufacturer sold the widgets to another buyer a week later for 11,000 the market price at the time. On May 1 the market price of the widgets dropped to 8000. The manufacturer’s cost to produce and deliver the widgets was 7000.

The manufacturer sued for breach of contract. At the time of trial, the market price of the widgets was 9000. The Court ruled in the manufacturer’s favork, and goudn that its sale of the widgets to the subsequent buyer was done in good father and in a commercially reasonable manner and that there was no incidental damages or expenses saves as a result of the breach.

Watamoutn of damages should the court award to the manufacturer?
a)nothing
]B) 1000
C) 2000
d) 3000
A

D
The court should award the manufacturer $3000 as lost profits. When a buyer anticipatorily breaches a contract for the sale of goods the seller’s basic damages are either the difference between the contract price and the market price as of the time for performance or the difference between the contract price and reslae price. If neither of these measures puts the seller in as good position as performance would have, and the seller is a lost volume seller, the seller may recover lost profits plus incidental damages. Here, the manufacturer, by virtue of the fact that it can manufacture as many widgets as it can sell and has ample investonry on hand, is a lost volume seller.

The difference between the contract price and the market price on May 1 (the time for perofmrance) was $2000. The difference between the contract price and the resale price is $0 because it sold them for more than the contract price. Neither fo those measures puts the manufacturer in as good a position as it would have been had the retailer performed under the contract. In that case, the manufacturer would have had $3000 profit from the sale to the retailer as well as a $4000 profit from the sale to the subsequent buyer.

The facts state that there are no incidental damages. Thus the manufacturer should be awarded the $3000 in lost profit as damages.

A) is incorrect because it is base don the difference between the contract price and the resale price. Because the manufacturer sold the widgets for more than the contract price, its damages would be 0 under this calculation. The manufacturer however is a lost volute seller and this measure of damages doe snot put the manufacturer is as good a position as it would have been had the retailer performed. Thus, the manufacturer is entitled lost profits.

124
Q

CAn an overbroad statute be enforced on someone who is NOT constitutionally protected? (ex. speech that involves imminent lawfulness).

A

NOPE. if it is overbroad that is the end of it.